Anda di halaman 1dari 87

www.monografias.

com

Contabilidad bsica
Estudio de la conversin del acto mercantil en hecho contable
1.
2.
3.
4.

Los negocios y sus transacciones


Las cuentas
El ciclo contable
Bibliografa

Los negocios y sus transacciones


1.1. Concepto de Contabilidad
El concepto de Contabilidad ha ido evolucionando con el tiempo, como un resultado de las necesidades de
desarrollo de la actividad comercial y del volumen creciente de las inversiones que esta ha ido requiriendo.
En sus orgenes, la contabilidad era slo una expresin de la observacin de los hechos acaecidos en el
intercambio de bienes, derechos y usos diferidos, pero es a partir de la utilizacin de la informacin contable
para analizar las relaciones lgicas del hombre con el flujo de la riqueza patrimonial para satisfacer sus
necesidades cuando se hace distinguible el objeto de la ciencia de la contabilidad.
Puede decirse que el concepto de contabilidad ha pasado por tres etapas, a saber: la Contabilidad como
una tcnica, la Contabilidad como una tecnologa y la Contabilidad como una ciencia.
LA CONTABILIDAD COMO UNA TECNICA:
La Contabilidad es el arte de coleccionar, resumir, analizar e interpretar datos financieros para determinar el
resultado del movimiento de los capitales en tiempos determinados y poder comprobar la exactitud e
integridad de los juicios y apreciaciones de ndole financiera que acerca de las circunstancias y de los
resultados se deduzcan.
LA CONTABILIDAD COMO UNA TECNOLOGIA
La Contabilidad es un sistema de informacin que permite recopilar, clasificar y registrar, de una forma
sistemtica y estructural, las operaciones mercantiles realizadas por una empresa, con el fin de producir
informes que analizados e interpretados permiten planear, controlar y tomar decisiones sobre la actividad de
la empresa.
LA CONTABILIDAD COMO UNA CIENCIA
La Contabilidad es una ciencia aplicada cuya finalidad es la medicin de la realidad cambiante en el tiempo
de los factores de riqueza en una entidad econmica, mediante mtodos especficos capaces de elaborar
una informacin financiera objetiva, de inters para los usuarios externos y valida para la toma de
decisiones internas.
Objetivos de la Contabilidad

Establecer en trminos monetarios, la cuanta de los bienes, las deudas y el patrimonio que
posee la empresa.

Llevar un control de los ingresos y egresos

Determinar las utilidades o perdidas obtenidas al finalizar el ciclo contable

Ejercer control sobre las operaciones del ente econmico y fundamentar la determinacin de
precios, tarifas y cargas tributarias.

Ofrecer en cualquier momento una informacin ordenada y sistemtica sobre el desenvolviendo


econmico y financiero de la empresa.

Servir de fuente fidedigna de informacin ante terceros (clientes, proveedores, inversionistas,


bancos y el Estado).

Facilitar la planeacin, organizacin y direccin de los negocios, no solo dando a conocer los
efectos de una operacin mercantil, sino tambin previendo situaciones futuras.
reas de Intervencin de la Contabilidad
Para su mejor estudio, en la Contabilidad pueden distinguirse tres campos de accin:

La Contabilstica o Tenedura de Libros, referida al arte de registrar en forma clara, ordenada


y sistemtica, transacciones comerciales u operaciones mercantiles. Hay que reconocer que en
cuanto exista un registro contable, la comprobacin y la revisin de los resultados permite
observar con gran facilidad los acontecimientos de la riqueza, por lo que a contabil stica le
puede ser atribuible una conceptualizacin de informacin contable.

La Finanza, es la interpretacin de las relaciones, en cualquier momento, entre los resultados


obtenidos y la situacin del capital invertido, considerando el tiempo transcurrido y siguiendo los

Para ver trabajos similares o recibir informacin semanal sobre nuevas publicaciones, visite www.monografias.com

www.monografias.com

procedimientos contables, establecidos por la ley. Para la bsqueda de la verdad acerca de los
fenmenos patrimoniales, tiene validez el anlisis y cotejo de las relaciones para sacar
conclusiones positivas acerca de las consecuencias de los hechos contables que se practiquen.

La Auditoria, consiste en el examen y comprobacin de la exactitud de la informacin contable


para dictaminar la calidad de las operaciones financieras efectuadas. El desarrollo de teoras de
las aplicaciones de naturaleza tecnolgica con el fin de producir un laudo arbitral capaz de
orientar cuestiones judiciales, administrativas, polticas, de peritazgos, auditorias, etc., hace
parte de un nivel superior del conocimiento, capaz de ampliar la aplicacin de los mtodos de
indagacin para opinar sobre una situacin evidenciada por los datos contables.
1.2. Concepto de Empresa
Segn del Artculo 25 del Cdigo de Comercio, se entiende por empresa toda actividad econmica
organizada para la produccin, transformacin, circulacin, administracin o custodia de bienes, o para la
prestacin de servicios.
Una organizacin puede ser con nimo de lucro o sin nimo de lucro. Las organizaciones con nimo de
lucro se denominan empresas o negocios. Las organizaciones sin nimo de lucro se denominan
Instituciones.
Dentro de las organizaciones existen tres tipos de factores necesarios para realizar sus operaciones, estos
son las personas que se encuentra representado por los propietarios, los gerentes, los administradores y
todos los empleados que laboran en los procesos administrativos y operativos de la empresa, el capital que
est constituido por los aportes que hacen los propietarios de la empresa y puede estar representado en
dinero en efectivo, en mercancas, en maquinaria, en muebles, en tecnologa o en cualquier otro aporte de
bienes y el trabajo que es la actividad que realizan las personas para lograr el objetivo primordial de la
empresa que puede ser la produccin de bienes, la comercializacin de mercancas o la prestacin de algn
servicio.
Desde el punto de vista econmico, el capital y el trabajo son intersustitubles. En la postmodernidad, se
distingue al personal de la empresa con el nombre de talento humano.
Clasificacin de las empresas
Las empresas se pueden clasificar segn los siguientes criterios: Por la actividad que realizan, por su
tamao, por el origen de su capital, su tamao, conformacin de su capital, por la explotacin y
conformacin de su capital, por los impuestos, por el nmero de propietarios, por la funcin social.
POR LA ACTIVIDAD
Extractivas: Son las empresas cuyo objetivo primordial es la explotacin de recursos que se encuentran en
el subsuelo, como ejemplo estn las empresas de petrleos, aurferas, de piedras preciosas y de otros
minerales.
Comerciales: Son las empresas que se dedican a la compra y venta de productos naturales, semielaborados y terminados a mayor precio del comprado, obteniendo as una utilidad. Un ejemplo de este tipo
de empresa es un supermercado.
Industriales: Son las dedicadas a transformar la materia prima en productos terminados o semi-elaborados
como las fbricas de telas, de muebles, de calzado, etc.
De Servicios: Son las que buscan prestar un servicio para satisfacer las necesidades de la comunidad, ya
sea de salud, educacin, transporte, recreacin, servicios pblicos, seguros y otros varios.
Agropecuaria: Son aquellas que explotan en grandes cantidades los productos de origen agrcola y
pecuario. Dentro de los ms comunes encontramos las granjas agrcolas, las granjas porcinas, avcolas,
apcolas, invernaderos, haciendas de produccin agrcola etc.
POR SU TAMAO
Grande: Es la de mayor organizacin, posee personal especializado en cada una de las reas de trabajo,
se observa una gran divisin del trabajo y las actividades mercantiles se realizan en un porcentaje elevado.
Comprende las empresas con ms de 200 trabajadores y ms de 15.001 SMMLV de capital
Mediana: En este tipo de empresa se observa una mayor divisin y delimitacin de funciones
administrativas y operacionales. La inversin y los rendimientos obtenidos son mayores que los de la
pequea empresa. Pueden tener entre 51 y 200 trabajadores y entre 5001 y 15000 SMMLV de capital.
Pequeas empresas: Son aquellas que manejan muy poco capital y poca mano de obra, se caracteriza
porque no existe una delimitacin clara y definida de funciones entre el administrador y/o el propietario del
capital de trabajo y los trabajadores; por ello existe una reducida divisin y especializacin del trabajo.
Pueden tener entre 11 y 500 trabajadores y un capital comprendido entre 501 y 5000 SMMLV.
Micro empresas: Su nmero de trabajadores esta comprendido entre 1 y 10, mientras que su capital se
halla entre 1 y 500 SMMLV. Las llamadas Famiempresas son un nuevo tipo de explotacin en donde la
familia es el motor del negocio convirtindose en una unidad productiva.

Para ver trabajos similares o recibir informacin semanal sobre nuevas publicaciones, visite www.monografias.com

www.monografias.com

POR EL ORIGEN DEL CAPITAL


Oficiales o Pblicas: Su capital proviene del Estado.
Privadas: Son aquellas en que el capital proviene de personas particulares.
De Economa Mixta: El capital proviene una parte del Estado y la otra de personas particulares.
POR LA EXPLOTACIN Y CONFORMACIN DE SU CAPITAL.
Multinacionales: En su gran mayora el capital es extranjero y explotan la actividad en diferentes pases del
mundo (globalizacin).
Grupos Econmicos: Estas empresas explotan uno o varios sectores pero pertenecen al mismo grupo de
personas o dueos.
Nacionales: El radio de atencin es dentro del pas normalmente tienen su principal en una ciudad y
sucursales en otras.
Locales: Son aquellas en que su radio de atencin es dentro de la misma localidad.
POR LOS IMPUESTOS
Personas Naturales Declarantes: Estn obligados a presentar declaracin del Impuesto de Renta y
Complementarios.
Sucesiones Ilquidas: En este grupo corresponde a las herencias y legados que se encuentran en proceso
de liquidacin.
Responsables del Rgimen Simplificado del Impuesto a las Ventas (IVA): Pertenecen al rgimen
Simplificado del IVA la sociedades y personas naturales que seale la las normas legales.
Para efectos fiscales los responsables del rgimen simplificado no estn obligados a llevar libros de
contabilidad, hallndose s obligados a llevar el libro fiscal de registro de operaciones diarias. El artculo 499
del E.T., precisa que pertenecen al rgimen simplificado del IVA, entre otras, las personas naturales
comerciantes de menores ingresos, sealamiento que independientemente de su calidad de comerciantes
es lo que permite atribuirles, para efectos fiscales, determinados derechos y obligaciones.
El Art. 499 del Estatuto Tributario dice: "Al Rgimen Simplificado del Impuesto sobre las Ventas pertenecen
las personas naturales comerciantes y los artesanos, que sean minoristas o detallistas; los agricultores y
ganaderos, que realicen operaciones gravadas, as como quienes presten servicios gravados, siempre y
cuando cumplan entre otras las siguientes condiciones:
- Que tengan mximo un establecimiento de comercio, oficina, sede, local o negocio donde ejercen sus
actividades.
- Que no sean usuarios aduaneros.
- Que no hayan celebrado en el ao inmediatamente anterior ni en el ao en curso -contratos de venta de
bienes o prestacin de servicios gravados por valor individual y superior a sesenta millones de pesos
($60.000.000)."
Es as como el artculo 506 del mismo ordenamiento, dentro de las obligaciones fiscales atribuidas a los
responsables del rgimen simplificado no formula exigencia legal alguna de llevar contabilidad en los
trminos y condiciones dispuestos por el Cdigo de Comercio.
Una interpretacin sistemtica de las normas reguladoras del impuesto al valor agregado permite precisar
claramente el manejo fiscal que la ley atribuye a los responsables del rgimen simplificado. Cuando el
pargrafo del artculo 437 del E.T., previene que, a las personas que pertenezcan al rgimen simplificado,
que vendan bienes o presten servicios, les est prohibido adicionar al precio suma alguna por concepto del
impuesto sobre las ventas, so pena de incumplir ntegramente las obligaciones predicables de quienes
pertenecen al rgimen comn, con ello est delimitando las obligaciones predicables del rgimen
simplificado no en razn de su calidad de comerciantes sino en razn de su actuar frente al rgimen fiscal,
lo que de suyo le imprime un tratamiento especial a este tipo de responsables.
Responsables del Rgimen Comn del Impuesto a las Ventas (IVA): Pertenecen al rgimen comn del
IVA la sociedades y personas naturales que seale la las normas legales. Art . 437 E.T.
Los comerciantes y quienes realicen actos similares a los de ellos y los importadores son sujetos pasivos,
responsables del Impuesto.
Son responsables del impuesto:
- En las ventas, los comerciantes, cualquiera que sea la fase de los ciclos de produccin y distribucin en la
que actan y quienes sin poseer tal carcter, ejecuten habitualmente actos similares a los de aquellos;
- En la venta de aerodinos, tanto los comerciantes como los vendedores ocasionales de stos;
- Los importadores;
- Los contribuyentes del Rgimen Comn del Impuesto sobre las Ventas, cuando realicen compras o
adquieran servicios gravados con personas pertenecientes al rgimen simplificado, por el valor del impuesto
retenido, sobre dicha transacciones.

Para ver trabajos similares o recibir informacin semanal sobre nuevas publicaciones, visite www.monografias.com

www.monografias.com

Empresario Gran Contribuyente: son contribuyentes que por el monto de los ingresos y patrimonio que
administran, la DIAN los clasifica como grandes contribuyentes, otorgndoles ciertas competencias
especiales tal como la auto retencin de IVA y otros impuestos.
POR EL NUMERO DE PROPIETARIOS
Empresas Individuales: denominadas tambin empresas unitarias o de propietario nico. En ellas, aunque
una persona es la duea, la actividad de la empresa se extiende a ms personas, quienes pueden ser
familiares o empleados particulares. Ejemplos: Almacn don PATRICIO, de propiedad exclusiva de don
PATRICIO ROMERO.
La empresa unipersonal, creada por la ley 222 de 1995, permite a los empresarios destinar una parte de
sus bienes a la realizacin de determinados negocios, dotndolos de personera jurdica y, por ende,
logrando que su responsabilidad quede limitada al monto del acervo asignado a la nueva empresa; y todo
esto podr alcanzarlo sin necesidad de acudir a otras personas que colaboren como socios reales o
simulados de la operacin.
Cuando por virtud de la cesin o por cualquier otro acto jurdico, la empresa unipersonal llegare a
pertenecer a dos o ms personas, deber convertirse en sociedad comercial. La nueva sociedad asumir,
sin solucin de continuidad, los derechos y obligaciones de la empresa unipersonal.
Cuando se utilice la empresa unipersonal en fraude a la ley o en perjuicio de terceros, el titular de las cuotas
de capital y los administradores que hubieren realizado, participado o facilitado los actos defraudatorios,
respondern solidariamente por las obligaciones nacidas de tales actos y por los perjuicios causados.
En lo que no est previsto por la presente Ley, se aplicar a la empresa unipersonal, las disposiciones
relativas a las sociedades comerciales y, en especial, las que regulan la sociedad de responsabilidad
limitada. As mismo, las empresas unipersonales estarn sujetas, en lo pertinente, a la inspeccin, vigilancia
o control de la Superintendencia de Sociedades, en los casos que determine el Presidente de la Repblica.
Sociedades: Se debe hacer claridad, que la legislacin colombiana no define como tal a las sociedades de
personas y a las sociedades de capital, sino que es una clasificacin que con el tiempo ha sido acogida por
muchos autores la cual se ha convertido en doctrina.
Se llaman sociedades de capital aquellas sociedades en las que lo que importa son los aportes econmicos,
las acciones y no las personas. En este tipo de sociedades a la administracin no importa en manos de
quien estn sus acciones. Entre las principales sociedades de capital, podemos mencionar:
Sociedad annima
Sociedad en comandita por acciones
Sociedad de economa mixta
En cambio, se consideran sociedades de personas, aquellas sociedades donde lo ms importante son las
personas que la conforman, es decir, sus socios y, por lo general, stas sociedades son administradas por
sus socios, conformados por miembros de una misma familia o por amigos muy cercanos. Entre estas, se
mencionan:
Cooperativas
Sociedad colectiva
Sociedad limitada
Sociedad en comandita simple
Sociedad unipersonal
La legislacin Colombiana para clasificar las sociedades comerciales, toma en consideracin las alcuotas
en que se divide el capital social; de ah que la codificacin comercial hace a lo largo de su articulado
referencia a tres formas especificas a saber: sociedades por partes de inters, por cuotas, y por
acciones, las que en su orden dicen de las fracciones en que est representado el capital de las
sociedades tpicas reconocidas a saber: colectivas las primeras, de responsabilidad limitad y en comandita
simple las segundas y, annimas y en comandita por acciones las ltimas.
El tema de la responsabilidad de los socios en los diferentes tipos de sociedades, difiere segn el tipo de
sociedad. En algunas, la responsabilidad de los socios se limita monto de sus aportes, y en otras, como en
las sociedades en comandita, se pueden presentar tanto la responsabilidad limitada como la ilimitada, en la
que los socios responden con su patrimonio solidaria e ilimitadamente.
Colombia cuenta ahora con un nuevo tipo de asociacin empresarial. Se trata de la Sociedad por Acciones
Simplificada (SAS), que permite su constitucin por una sola persona natural o jurdica y creada por tanto
mediante acto unilateral, en el que no existe un lmite mximo ni uno mnimo y por lo tanto hace posible
tener sociedades unipersonales o empresas con multiplicidad de accionistas, sin lmite en el nmero.
La SAS ha sido diseada para facilitar trmites, reducir costos y tener ms flexibilidad en las organizaciones
colombianas. *La ley 1258 de 2008 que dio vida a esta tipologa empresarial reza en sus tres primeros
artculos: La sociedad por acciones simplificada podr constituirse por una o varias personas naturales o

Para ver trabajos similares o recibir informacin semanal sobre nuevas publicaciones, visite www.monografias.com

www.monografias.com

jurdicas, quienes slo sern responsables hasta el monto de sus respectivos aportes. Salvo lo previsto en
el artculo 42 de la presente ley, el o los accionistas no sern responsables por las obligaciones laborales,
tributarias o de cualquier otra naturaleza en que incurra la sociedad.
Cuando una sociedad se disuelva por la reduccin del nmero de socios a uno, podr, sin liquidarse,
convertirse en empresa unipersonal, En este caso, la empresa unipersonal asumir, sin solucin de
continuidad, los derechos y obligaciones de la sociedad disuelta.
La legislacin colombiana mediante la ley 1014 de 2006, haba creado las sociedades unipersonales como
sociedades iguales a todas las dems, salvo en la circunstancia de tener un socio nico. Por su parte, el
Articulo 46 de la ley 1258 de 2008 estableci para las sociedades unipersonales la obligacin de
transformarse en SAS durante los seis meses siguientes a la expedicin de esta ley. Sin embargo, segn
decisin de la Corte, las sociedades unipersonales constituidas durante el lapso de emisin de la ley 1014
de 2006 y la ley 1258 de 2008 no estn obligadas a convertirse en SAS.
1.3. La Actividad Mercantil
La actividad mercantil o actividad comercial se refiere a los actos de comercio legalmente calificados como
tales y a las relaciones jurdicas derivadas de la realizacin de estos, en trminos amplios es la rama de
actividad econmica referida al ejercicio del comercio.
"Se denominan comerciantes a las personas que profesionalmente se ocupan en alguna de las actividades
que la ley considera mercantiles. La calidad de comerciante se adquiere aunque la actividad mercantil se
ejerza por medio de apoderado, intermediario o interpuesta persona". (Art. 10 C. de Co). Las personas que
ejecuten ocasionalmente operaciones mercantiles no se consideran comerciantes, pero estarn sujetas a
las normas comerciales en cuanto a dichas operaciones. (Art. 11 C. de Co).
Se presume que una persona ejerce el comercio en los siguientes casos:

Cuando se halla inscrita en el Registro Mercantil.

Cuando tenga un establecimiento de comercio abierto y

Cuando se anuncia al pblico como comerciante por cualquier medio. (Art. 13 C. de Co).
OBLIGACIONES DE LOS COMERCIANTES

Matricularse en el Registro Mercantil.

Inscribir en el Registro Mercantil todos los actos, libros y documentos respecto de los cuales la
ley exige esa formalidad.

Llevar contabilidad regular de sus negocios conforme a las prescripciones legales.

Conservar, con arreglo a la ley, la correspondencia y dems documentos relacionados con sus
negocios o actividades.

Denunciar ante el juez competente la cesacin en el pago corriente de sus obligaciones


mercantiles.

Abstenerse de ejecutar actos de competencia desleal. (Art. 19 C. de Co).


REQUISITOS LEGALES PARA EL EJERCICIO DE LA ACTIVIDAD COMERCIAL.
Los comerciantes legalmente establecidos, o sea matriculados en la Cmara de Comercio, para poder
ejercer su actividad mercantil, deben tramitar los siguientes documentos.

El Registro nico Tributario (RUT) expedido por la Direccin de Impuestos y Aduanas


Nacionales (DIAN).

La Matricula de Industria y Comercio, expedida por la Tesorera Municipal.

La Patente de Sanidad, expedida por la Secretaria de Salud Publica.

El Certificado del Cuerpo de Bomberos.

Los certificados expedidos por las oficinas de Planeacin Municipal, Obras Publicas, para lo de
su requerimiento, de acuerdo con la actividad u objeto social del comerciante.

Paz y Salvo de SAYCO y ACINPRO.


Transaccin Comercial
Se entiende por transaccin comercial todo hecho o acto econmico que implique la compra o venta de un
bien o servicio. Una transaccin comercial es todo aquello que ocurre cuando hay una adquisicin de bienes
o servicios de forma onerosa con destino a enajenarlo en igual forma.
Las transacciones comerciales, tambin llamadas financieras, producen documentos fuentes de
informacin, los cuales constituyen la base de la contabilidad; entre estos documentos cabe destacar los
cheques de banco, las facturas extendidas y las pagadas, los recibos de caja, etc.. La informacin contenida
en estos documentos es trasladada a los libros contables, el diario y el mayor. En el libro diario (o Bitcora
del negocio) se reflejan todas las transacciones realizadas por la empresa, mientras que en el mayor (o
Control comercial) se reflejan las transacciones que afectan a las distintas partidas contables, por ejemplo,
caja, bancos, clientes, proveedores, etctera; esto es lo que llamamos asientos contables, por medio de los

Para ver trabajos similares o recibir informacin semanal sobre nuevas publicaciones, visite www.monografias.com

www.monografias.com

cuales resumimos todas las operaciones mercantiles en los comprobantes de diario y registramos en los
libros de contabilidad.
Las transacciones son como las necesidades que deben cumplir las organizaciones. Las transacciones
comerciales ms comunes, son:

Aportes de Capital

Consignacin en Entidades Financieras

Compra de Mercancas al Contado

Compra de Mercancas a Crdito

Compra a Crdito de activos diferentes a mercancas

Ingresos recibidos por Anticipado

Prstamos Recibidos por la empresa

Gastos causados por Pagar

Venta de Mercancas al Contado

Ingresos corrientes

Venta de Mercancas a Crdito

Cargo o Dbito a Proveedores

Abono o Crdito de Clientes

Gastos

Cargo a Obligaciones Bancarias

Gastos pagados por Anticipado

Prstamos Concedidos por la empresa


1.4. Los Hechos Contables
Como ya ha sido mencionado, las transacciones son hechos o actos econmicos. De acuerdo a su
naturaleza, los hechos econmicos deben ser apropiadamente tipificados, siguiendo para ello un plan
contable previamente establecido por la empresa o institucin y una metodologa de registro de la
informacin, de tal manera que se registren de modo apropiado en las cuentas adecuadas. Es as, como los
hechos econmicos son transformados en hechos o actos contables.
Los hechos contables pues, inician tras la incorporacin formal en la contabilidad de aquellos sucesos
puntuales que hacen variar las cuentas de la empresa, pero es a partir de la utilizacin de la informacin
contable para analizar los cambios patrimoniales debido a los movimientos de la riqueza involucrada por los
emprendimientos humanos, cuando se hacen distinguibles como el objeto de la ciencia de la contabilidad.
El ciclo contable es el proceso que debe seguirse para transmitir correctamente la informacin contable a
los usuarios de dicha informacin. Las autoridades econmicas profieren normas bsicas y normas tcnicas
para regular el ciclo contable y garantizar as, que todos los hechos econmicos sean reconocidos (Art. 46
Decreto 2649 de 1993).
En este contexto el ejercicio profesional de la Contadura en Colombia se rige por la Ley 43 de 1990
mientras que la practica contable la regula el Decreto 2649 de 1993. El Artculo 1. del Decreto 2649 de
Diciembre 29 de 1993, expresa: "De conformidad con el artculo 6 de la ley 43 de 1990, se entiende por
principios o normas de contabilidad generalmente aceptados en Colombia, el conjunto de conceptos
bsicos y de reglas que deben ser observados al registrar e informar contablemente sobre los asuntos y
actividades de personas naturales o jurdicas".
Apoyndose en ellos, la contabilidad permite identificar, medir, clasificar, registrar, interpretar, analizar,
evaluar e informar, las operaciones de un ente econmico, en forma clara, completa y fidedigna.
LA PARTIDA DOBLE
La contabilizacin de los hechos contables se realiza basndose en el mtodo de la Partida Doble, el cual
puede enunciarse as: todo hecho contable afecta como mnimo a dos partes o cuentas
Esto significa que para anotar cualquier transaccin, por lo menos, es necesario efectuar una doble
anotacin en dos partes contrapuestas (causa y efecto) que se utilizan en todo hecho contable
Por ejemplo, si una persona natural inicia un negocio cualquiera aportando $ 70.000.000, esta primera
transaccin que podremos llamar de Aportes de Capital dar lugar al siguiente registro contable en el Libro
de Diario:
Fecha
24/08/20xx

Cdigo

Nombre de la Cuenta
Caja
Capital de Personas Naturales

Debe
70.000.000

Haber
70.000.000

Para ver trabajos similares o recibir informacin semanal sobre nuevas publicaciones, visite www.monografias.com

www.monografias.com

Aqu, se llama Caja al lugar, parte o cuenta donde se va a depositar momentneamente el dinero recibido,
mientras que se llama Capital de personas naturales al lugar, parte o cuenta que entrega la suma de dinero
aportada, quedando el negocio con la obligacin de responderle a la persona aportante por tal suma de
dinero para cuando la quiera retirar.
Como puede verse, el mtodo de la Partida doble se basa en los siguientes principios:

En todo hecho contable o transaccin, siempre hay un deudor o varios deudores, por el valor de
la operacin y, un acreedor o varios acreedores por el mismo importe.

En la contabilizacin de toda transaccin, el deudor es el elemento que recibe y, acreedor es


aquel otro elemento que entrega.
Ahora, para ser ms didcticos, podemos adoptar la analoga existente entre el esquema del manejo de una
cuenta y la letra T del alfabeto:

NOTA: Observe Usted como aqu se ha considerado ms conveniente anotar los movimientos y el saldo de
la cuenta en la parte superior de la T, En el modo clsico y muy del criterio personal de los contadores, los
movimientos (dbito y crdito) y, consecuentemente el saldo (dbito o crdito), son anotados en la parte
inferior de la T. La conveniencia pues, radica en el permanente despliegue de estos valores muy cercanos al
encabezamiento de la T.
Las partes de una cuenta son:
el Nombre de la cuenta, el cual se anota en el encabezamiento de la T.
el Nombre de las columnas de la T. En la parte superior de la columna izquierda, se anota la palabra
DEBE y en la de la columna derecha se anota la palabra HABER, respectivamente. La cuenta se
divide en 2 partes; una de ellas, se destina a recoger los aumentos de valor (si es cuenta de Activo,
Gasto o Costo, nos estamos refiriendo a columna de la izquierda) y la otra, recoge las
disminuciones de valor (si es cuenta de Activo, Gasto o Costo, nos referimos a la columna de la
derecha).
los registros que se hagan en la columna del DEBE, son llamados DEBITO o CARGO, mientras que
los registros en la columna del HABER, se llaman CREDITO o ABONO.
la suma de los valores registrados en el DEBE de una cuenta se denomina MOVIMIENTO DEBITO,
mientras que la suma de los valores registrados en el HABER se llama MOVIMIENTO CREDITO.
la diferencia entre el movimiento debito y el movimiento crdito se llama SALDO DE LA CUENTA.
Una cuenta tendr SALDO DEBITO cuando el movimiento debito sea mayor que el movimiento
crdito y, en contrario, una cuenta tendr SALDO CREDITO cuando el movimiento crdito sea
mayor que el movimiento debito.
Ahora observando el registro del mismo ejemplo en cuentas T, encontraremos que tambin se cumple la
norma de la partida doble; la cual se refiere a que un asiento afecta a dos cuentas como mnimo, esto es:

Para ver trabajos similares o recibir informacin semanal sobre nuevas publicaciones, visite www.monografias.com

www.monografias.com

Sea otro ejemplo. Supongamos que al da siguiente, como una medida de control, la empresa decida
consignar el dinero recibido en una cuenta de ahorros o una cuenta corriente en un banco . En este
caso podremos llamar a esta segunda transaccin como Consignacin en entidades financieras, la cual
dar lugar al siguiente registro en libros:
Fecha
25/08/20xx

Cdigo

Nombre de la Cuenta
Bancos
Caja

Debe
70.000.000

Haber
70.000.000

En este caso, llamamos Banco al lugar, parte o cuenta donde se van a guardar los dineros recibidos,
mientras que la parte o cuenta que entrega el dinero es la que anteriormente habamos denominado como
Caja. Es por eso que ahora, el registro en Banco es un dbito mientras que el registro en Caja es un
crdito.
Volvamos a observar esto mismo con ayuda de las cuentas T. En este caso, identificamos el asiento
anotndole un parntesis con el nmero de la transaccin, esto es (2):

Observamos ahora que como resultado de las dos transacciones, el lugar, parte o cuenta que hemos
llamamos Caja, contiene dos registros correspondientes a un igual nmero de transacciones en que ha
participado. En este ltimo registro (identificado con el nmero 2 entre parntesis), vemos que tambin se
cumple la norma de la partida doble.
EL PLAN UNICO DE CUENTAS
Un Plan nico de Cuentas es el listado de cuentas adoptado por una empresa con su respectiva descripcin
y los procedimientos para su aplicacin contable.
Un Plan de Cuentas esta estructurado sobre la base de una codificacin que comprende niveles o
categoras de clasificacin, as:
Primer digito: CLASE
Segundo digito: GRUPO
Tercer y cuarto digito: CUENTA
Quinto y sexto digito: SUBCUENTA
Sptimo y octavo digito: AUXILIAR
Segn el Decreto 2894 de 1994, el Ministerio de Hacienda determino el Catalogo de Cuentas Contables que
debe usarse en el territorio colombiano, agrupndolas del siguiente modo:
1- ACTIVOS
2- PASIVOS

Para ver trabajos similares o recibir informacin semanal sobre nuevas publicaciones, visite www.monografias.com

www.monografias.com

345678-

PATRIMONIO
INGRESOS
GASTOS
COSTOS DE VENTAS
COSTOS DE PRODUCCION
CUENTAS DE ORDEN

LOS ASIENTOS CONTABLES


Un asiento contable es el registro de las operaciones mercantiles, tomando como base las cifras
presentadas en los soportes de contabilidad (recibo de caja, boleta de consignacin, etc.) En el Registro
Contable se Pueden Presentar dos Tipos de Asientos
Asiento Simple: cuando solamente se afectan dos cuentas correspondientes a un dbito y a un crdito.
Ejemplo: el da 1 de Septiembre, el Almacn de Electrodomsticos El Yiyo, compra al contado, 10
cmaras fotogrficas por $ 15.000.000.
En esta transaccin de Compras de Mercancas al Contado, las cuentas afectadas son dos: DE
MERCANCIAS, cuenta de Compras que por aumentar, se debita; y BANCOS, cuenta de Activos que por
disminuir, se acredita. En forma escrita, el registro contable toma la siguiente configuracin:
Fecha
01/09/20xx

Cdigo

Nombre de la Cuenta
De Mercancas
Bancos

Debe
15.000.000

Haber
15.000.000

Asiento Compuesto: cuando en el registro se afectan ms de dos cuentas, pueden ser una cuenta deudora
y dos o ms cuentas acreedoras, o dos o ms cuentas deudoras y una acreedora. Ejemplo: si en la
compra anteriormente mencionada se paga la tercera parte al contado y las otras dos terceras
partes se aceptan a crdito por 30 das.
En esta transaccin de Compras de Mercancas a Crdito y al Contado, las cuentas afectadas son tres: DE
MERCANCIAS, cuenta de Compras que por aumentar, se debita; PROVEEDORES, cuenta de Pasivos que
por aumentar, se acredita y, BANCOS, cuenta de Activos que por disminuir, se acredita. En forma escrita, el
asiento sera entonces:
Fecha
01/09/20xx

Cdigo

Nombre de la Cuenta
De Mercancas
Proveedores
Bancos

Debe
15.000.000

Haber
10.000.000
5.000 000

Lo que en cuentas T, puede escribirse:

De lo anterior, se desprenden las siguientes dos propiedades del principio de la partida Doble:
En todo hecho contable, la suma del valor adeudado a uno o varios elementos patrimoniales, ha de
ser igual a la suma del valor abonado a otros.
En cualquier momento, la suma del debe ha de ser igual a la suma del haber.
En el mtodo de la partida doble, el efecto del ejercicio contable se puede hallar de dos maneras: a)
Balanza = Activo Pasivo y, b) Resultado = Ingresos Gastos.
LA ECUACIN CONTABLE

Para ver trabajos similares o recibir informacin semanal sobre nuevas publicaciones, visite www.monografias.com

www.monografias.com

La ecuacin contable se puede enunciar diciendo: El total del Activo (A) es igual a la suma del Pasivo (P)
mas el Patrimonio (Pn )
Se llaman ACTIVOS a los recursos que posee negocio. Son ejemplos de activos el efectivo en caja, los
terrenos, los edificios y los equipos de trabajo.
Son PASIVOS las obligaciones que recaen sobre estos recursos del negocio. Suele decirse que reflejan los
derechos de los acreedores.
El PATRIMONIO refleja el compromiso del negocio con sus propietarios y representa el derecho o inversin
de los dueos del negocio, que consiste generalmente, en sus aportaciones, ms las utilidades retenidas (o
perdidas), ms otros tipos de supervit, como pueden ser las revaluaciones o donaciones. El Patrimonio
tambin recibe el nombre de Capital Contable, para designar la diferencia que resulta entre el activo y el
pasivo de una empresa.
As, llegamos a siguientes expresiones fundamentales del sistema contable:
ACTIVO = PASIVO + PATRIMONIO.
ACTIVOS + GASTOS = PASIVO + INGRESOS
VARIACION DEL PATRIMONIO = NUEVOS APORTES + UTILIDADES - RETIROS
1.5. Los Estados Financieros
La contabilidad representa la situacin inicial del patrimonio de la empresa y las variaciones que ste haya
experimentado a lo largo del ejercicio econmico, procediendo al final a ordenar la informacin para calcular
el beneficio o prdida del ejercicio y para informar sobre el valor del patrimonio en ese momento. La
informacin as obtenida se resume a una fecha o por un perodo determinado en unos documentos
esencialmente numricos tambin llamados ESTADOS CONTABLES que presentan la situacin financiera
de una empresa, los resultados obtenidos en un perodo determinado y el comportamiento del efectivo.
EL BALANCE GENERAL
El Balance General muestra la naturaleza y cantidad de todos los activos posedos, naturaleza y cantidad
del pasivo, tipo y cantidad de inversin patrimonial de los propietarios de un negocio. Tiene por objeto rendir
un claro y preciso informe a las partes interesadas en la empresa sobre la situacin de la misma al final de
un ao comercial. Los rubros del Balance General se clasifican en:
Activos: suma de bienes, recursos y derechos que posee la empresa
Pasivos: obligaciones o compromisos que ha adquirido la empresa hacia terceros
Capital o patrimonio: diferencia del activo y el pasivo, representa la inversin o el aporte de socios y
empresarios sobre los cuales puedan tener derecho.
EL ESTADO DE RESULTADOS
El estado de resultados, tambin llamado Estado de Ingresos y Gastos o de Ganancias y Prdidas. Se
elabora con el fin de saber cules fueron las utilidades en un perodo determinado. Este Estado presenta el
exceso de los ingresos sobre los gastos lo que da origen a la utilidad neta y al exceso de los costos y los
gastos sobre los ingresos se llama prdida neta. Si el resultado es utilidad neta el capital contable de la
empresa aumenta, si es una prdida disminuye. El xito de un negocio se juzga principalmente a travs de
sus ganancias, no nicamente por cuanta, sino tambin por la tendencia que manifiestan. Los rubros del
Estado de Resultados se clasifican en:
Fuentes del ingreso de las operaciones
Costos y gastos comerciales de operaciones
Prdidas y ganancias operativas durante el perodo.
. Provisiones para Reservas e Impuestos.
. Utilidades o prdidas netas que causan efectos sobre el patrimonio de la empresa
EL ESTADO DE FLUJO DE EFECTIVO
El Estado de Flujo de Efectivo puede definirse como un documento que presenta el impacto que tienen las
actividades operativas de inversin y de financiamiento de una empresa sobre sus flujos de efectivo a lo
largo de un perodo contable o fiscal. Su propsito es mostrar la informacin pertinente sobre los cobros y
pagos de efectivo de una empresa durante un perodo. Se utiliza principalmente para evaluar la habilidad de
la empresa generando futuros flujos de efectivo para cumplir con sus necesidades de financiamiento;
evaluar las razones de la diferencia entre las utilidades netas y los cobros y pagos de efectivo asociado. Se
prepara analizando todo el movimiento concerniente a las entradas y salidas de efectivos especficos. Los
rubros se clasifican siguiendo las siguientes actividades:
Actividades operativas

Para ver trabajos similares o recibir informacin semanal sobre nuevas publicaciones, visite www.monografias.com

www.monografias.com

a) Efectivo recibido de clientes.


b) Efectivo pagado a proveedores.
c) Efectivo pagado por intereses, etc.
Actividades de inversin
a) Ventas de activos fijos.
b) Adquisicin de activos fijos.
Actividades de financiamiento.
a) Prstamos pblicos.
b) Emisin de acciones.
c) Pago de dividendos.
1.6. La Apertura de los Negocios
EL INVENTARIO DE BIENES Y OBLIGACIONES
Un Inventario de Bienes y Obligaciones es un documento privado, posiblemente confeccionado por no
contadores, que representa el primer informe que se realiza antes de constituir formalmente un negocio. El
Inventario de Bienes y Obligaciones consiste en una relacin ordenada, clasificada y valorada de los Bienes
y Derechos con que comienza el negocio (Activos), las Obligaciones y Derechos del negocio que son ajenas
(Pasivos) y los Aportes o Derechos de los propietarios (Patrimonio o Capital Contable)
Las partes del Inventario de Bienes y Obligaciones son:
El Encabezamiento, el cual comprende la Razn Social conque se le piensa llamar al negocio, el
nombre del Documento de que se trata y la fecha de realizacin;
el Cuerpo que incluye tres subpartes o ttulos llamados Activos (A), Pasivos (P) y Capital Contable
(Pn ), cuyos valores cumplen con la ecuacin patrimonial A= P + P n;
las Firmas Responsables del Dueo, Gerente o quien represente legalmente a estos y dos testigos.
En todos los casos la firma del Gerente debe ser autenticada ante Notario Pblico y cuando se trate
de sociedades, uno de los testigos debe ser un Contador Pblico con matrcula Profesional vigente.
Las reglas ms comnmente aplicadas son las siguientes:
Los bienes ms parecidos entre si, segn las funciones que desempean en el negocio, se agrupan
y se les asigna un nombre que represente lo ms posible a dicho grupo de bienes, el cual no tiene
porque coincidir con los nombres utilizados en el Plan de Cuentas
Tanto los bienes componentes del ACTIVO como del PASIVO deben comprobarse por un recuento
fsico real, de peso, cantidad o medida de cada una de las unidades que lo integran
La valorizacin de esas unidades ha de ser, por regla general, la de su costo, pero si en el momento
de hacer el inventario se encontraren unidades a un costo mayor del que hay en el mercado ese
mismo da y tal diferencia representare una disminucin apreciable, entonces, esas unidades
debern valorizarse por el precio (ms bajo, o sea el) del mercado.
Todos los bienes y obligaciones inventariadas deben ser de propiedad exclusiva o endosadas a
nombre de el (los) empresario (s) fundador (es),
No deben incluirse las mercancas deterioradas, edificaciones destruidas y abandonadas, o
cualquier otro bien carente de un valor real
.Deben excluirse del inventario las mercancas recibidas en comisin o en consignacin, as como
aquellas facturadas y no enviadas a los clientes, o pagos anticipados sobre contratos de compra, o
esas otras mercancas que han sido remitidas en consignacin a clientes y de las cuales no se haya
recibido finiquito de ventas.
Debe confeccionarse un informe especial pormenorizado de los rubros con las especificaciones de
cantidad, calidad, nombre completo, precio por unidad y valor respectivo, con sus valores
totalizados debidamente clasificados
Ejemplo:
El Activo del negocio mercantil Pintura Arte y Decoracin EU est compuesto de $ 10.000.000 en Efectivo, $
220.000.000 en Bienes Races de cuyo avalo emitido por la Lonja de Propiedad Raz de Barranquilla se
incluye una Bodega de 250 mt2 a 500.000 $/mt2 cubierta en estructura metlica de dos aguas para
almacenamiento de pinturas y un Lote de terreno de 1000 mt2 ubicado en Calle 17 N 30-20 de la ciudad de
Barranquilla; 50 Galn de Pintura al aceite, marca Terinsa, en diferentes colores a $ 51.105 / Gl y 8 Tambor
de Base acrlica para pinturas al agua, marca Barrio Montes a $ 3.750.000 / Tm. El Pasivo consta de
Facturas por Pagar por Mercancas recibidas a Crdito de Terin S.A., Fact N 2830, con vencimiento en
30/09/xx por $ 1.750 y de Daro Barrionuevo, Fact N 157, con vencimiento en 12/04/xx por $ 17.000.000;
en Letras Aceptadas por Pagar $ 8.000.000 a Edinson Umaa, para el 31/12/xx, tenedor actual Asesoras y

Para ver trabajos similares o recibir informacin semanal sobre nuevas publicaciones, visite www.monografias.com

www.monografias.com

Cobranzas Ltda. y .$ 5.000.000 a Hernando Celedn, Letra girada por Pagar para el 01/05/xx Adems, pesa
una hipoteca sobre el inmueble del Lote de Terreno del Barrio Rebolo a nombre de Abogados Asociados.
Cmo ser el Informe del Inventario de Bienes y Obligaciones y cul es el Capital Contable?

EL RESUMEN O COMPROBANTE DE DIARIO


El Resumen de Diario es un documento contable elaborado por el empresario que le permite hacerse una
idea sobre la situacin de las operaciones de la empresa realizadas en el periodo. Se les denomina tambin
Comprobante de Diario.

Para ver trabajos similares o recibir informacin semanal sobre nuevas publicaciones, visite www.monografias.com

www.monografias.com

Un Resumen o Comprobante de Diario es un formato que se utiliza para visualizar la lista del total de los
dbitos y de los crditos de las cuentas que han tenido movimientos, refleja la contabilidad de una empresa
en un determinado periodo.
El objetivo final de este documento es comprobar que no existen asientos descuadrados en la contabilidad.
Ahora bien, esto no significa que todo est en orden, sino nicamente que la contabilidad est cuadrada.
Por ejemplo: hemos podido pagar a un proveedor y habrselo anotado a otro. En este caso el Comprobante
de Diario saldr correcto y sin embargo la contabilidad estar mal realizada.
Por lo general, la elaboracin del Comprobante de Diario no es de carcter obligatorio para el empresario,
aunque es muy recomendable para que este pueda conocer con precisin el estado de sumas debito y
crdito de cada ejercicio de operaciones contables sin necesidad de arrastrar errores hasta la elaboracin
de las cuentas anuales.
Para su elaboracin se siguen los siguientes pasos:
1. Se escribe en el encabezamiento: el nombre de la empresa, el nmero del comprobante, la fecha de
elaboracin y el tipo de resumen, en 3 lneas, centradas.
2. Se trasladan las anotaciones del libro Diario que hayan tenido lugar durante el ejercicio en un
borrador en metodologa de cuentas T.
3. Se obtienen para cada cuenta las sumas de las anotaciones, tanto en el debe como en el Haber.
4. Se trasladan estas sumas (movimientos) al formato de Comprobante de Diario en el orden numrico
que indica el cdigo de cuentas. Segn la importancia de ciertas cuentas (Caja, Bancos, Clientes,
Proveedores, etc.) puede convenir discriminar los sumandos originarios de cada valor.
5. Se suman las columnas Debe y Haber las cuales deben arrojar sumas iguales.
EL BALANCE GENERAL DE APERTURA DE LA CONTABILIDAD
El Balance de Apertura es el estado contable utilizado para registrar el primer asiento tanto en el Libro
Diario, como en el Libro Mayor y el Libro de Inventarios y Balances. El Balance de Apertura se construye en
formato de Comprobante Diario (Ver pasos 3 y 4 del procedimiento anteriormente descrito para la
elaboracin de Comprobantes de Diario) con el que tambin se inicia la numeracin consecutiva, a partir de
la informacin suministrada por el Inventario de Bienes y Obligaciones. Este Inventario de Bienes y
Obligaciones, se constituye como el soporte contable del Comprobante de Diario N 1.
En un Balance de Apertura, tiene lugar una reclasificacin de los grupos de bienes reconocidos en el
Inventario de Bienes y Obligaciones, de tal modo que puedan recibir el nombre que tienen las cuentas
correspondientes de Decreto 2649 de 1990 (PUC). El siguiente es la ilustracin del Balance de Apertura
Contable:

Para ver trabajos similares o recibir informacin semanal sobre nuevas publicaciones, visite www.monografias.com

www.monografias.com

Lo que en cuentas T (representando lo que ocurre en el Libro Mayor), ser:

1.6. Evaluacin
1.6.1. Problemas Resueltos
1- Por medio de una cuenta T, calcule el saldo dbito o crdito de la cuenta
a) CAJA, cuyos registros dbito son. $ 130.000, 110.000, 170.000, 90.000 y sus registros crdito
son: $ 120.000, 120.000.150.000 y 90.000
b) Proveedores NACIONALES, cuyos registros crdito son: $ 280.000, 190.000, 420.000 y
dbitos: $ 160.000, 230.000.
Solucin: Primero se calculan los movimientos y luego el saldo respectivo, as:

Para ver trabajos similares o recibir informacin semanal sobre nuevas publicaciones, visite www.monografias.com

www.monografias.com

Como puede observarse, la CAJA presenta un saldo dbito de $ 20.000.000, mientras la cuenta de
Proveedores NACIONALES presenta un saldo crdito de $ 500.000.000
2- Si una empresa posee un activo total de $ 200.000.000 y su pasivo asciende a $ 60.000.000,
entonces su patrimonio lquido es de $ 140.000.000.
a) Cul es el capital propio?.
b) Cul es el capital de terceros?
c) Cul es el capital contable?
Solucin: Aqu se debe aplicar la ecuacin patrimonial. El Capital Contable o Patrimonio, designa
la diferencia que resulta entre el activo y el pasivo de una empresa y representa la inversin de los
propietarios, que consiste generalmente, en sus aportaciones, ms las utilidades retenidas (o
perdidas), ms otros tipos de supervit, como pueden ser las revaluaciones o donaciones.
A P = Pn
200.000.000 60.000.000 = 140.000.000
a) Capital propio : $ 140.000.000
b) Capital de terceros: $ 60.000.000
c) Capital Contable. $ 140.000.000
3- Una empresa fue constituida con un capital social de $ 10.000.000, representado por los siguientes
guarismos: Caja $ 2.500.000, Muebles y enseres: $ 4.500.000, Salarios por pagar $ 800.000,
Inventario de Mercancas: $ 2.350.000, Construcciones e instalaciones: $ 5.000.000. Qu cantidad
de ganancia o perdida ha tenido la empresa hasta el momento?
Solucin: En primer lugar debe confeccionarse un inventario de bienes y obligaciones, as:

Para ver trabajos similares o recibir informacin semanal sobre nuevas publicaciones, visite www.monografias.com

www.monografias.com

Al restar del Patrimonio (Capital Contable) las aportaciones del empresario (Capital de Personas
Naturales), obtendremos las utilidades de ejercicios anteriores por retirar (o prdidas por reponer),
ms otros tipos de supervit, como pueden ser las revaluaciones o donaciones, que en este caso se
toman como no existentes:
Utilidades por retirar = Patrimonio Capital de Persona natural
= 13.550.000 10.000.000
= 3.550.000
4- David Cervantes es propietario y administrador de Asesoras Excelencia SAS. Al 31 de Diciembre la
empresa tena activos por $ 452.000.000 y pasivos por $ 158.200.000. Explique sus respuestas a
las siguientes preguntas:
a) Cul es el capital contable de David al 31 de diciembre del mismo ao?
b) Si aumentan sus activos en $ 50.000.000 y sus pasivos en $ 15.000.000, adems de que no
hubo retiros ni nuevas inversiones, cual ha sido su prdida o utilidad durante el ao?
c) Si aumenta sus activos en $ 9.000.000 e incrementa sus pasivos en $ 22.000.000, adems de
que no hubo retiros ni nuevas inversiones, cual ha sido su prdida o utilidad durante el ao?
d) Si el 31 de diciembre del mismo ao, sus activos valen $ 376.830.000 y los pasivos $
183.030.000 adems de que no hubo retiros ni nuevas inversiones, cual ha sido su prdida o
utilidad durante el ao?
Solucin: Aqu se debe aplicar la ecuacin patrimonial: A P = K, para posteriormente considerar la
variacin patrimonial.
a) 452.000.000 158.200.000 = 293.800.000
b) (452.000.000 + 50.000.000) (158.200.000 + 15.000.000) = 328.800.000
Utilidades = Variacin del Patrimonio Nuevos Aportes + Retiros
= (328.800.000 293.800.000) 0 + 0 = 35.000.000
c) (452.000.000 - 9.000.000) (158.200.000 + 22.000.000) = 280.800.000
Perdidas = 280.800.000 293.800.000 = - 13.000.000
d) (376.830.000 183.030.000) 293.800.000 = - 100.000.000

Para ver trabajos similares o recibir informacin semanal sobre nuevas publicaciones, visite www.monografias.com

www.monografias.com

5- El Activo del negocio del seor Johnny Pies calientes, comerciante al por menor establecido en
Barranquilla en la Calle 69 N 32-76, en Viernes 1 de Enero est compuesto de $ 1.440.000 en
dinero (billetes y monedas); $ 24.715.384 depositados en el Banco de Bogot y $ 15.023.077 en el
Banco Mercantil; 4.320 metros de Tela Estampada Coltejer a $ 4.375 cada uno, 1440 m. de Dril
Naval blanco a $ 22.430 c/u, 1200 pares de Media Punto Blanco a $ 3850 c/u, 2.152 pares de Media
GEF para nio a $ 3.000 c/u, 20 caja x 24 de Pauelos Pyramide a $ 53.000 c/u y 36 piezas x 1m.
de Fondo para sillas Jacquard a $ 23.750 c/u; 4 Estante Metlico Moderna a $ 2.225.000 c/u, 1
Vitrina-mostrador grande Cruz y Can de $ 2.000.000 1 Registradora Canon N 4734 de $
1.500.000 y 1 Escritorio con su silla de $ 1.665.000; adems existen facturas pendientes de pago a
nombre de Pedro Yudex por $ 5.820.000, Luis Puntilla por $ 2.900.000 y Elber Rosales por $
1.700.000. El Pasivo consta de Facturas por Pagar por Mercancas recibidas a Crdito de Torres y
Torres Ltda. de Cali segn factura N 234 a 90 das por $ 7.753.846, Textileros Asociados S.A de
Medelln s/fra. N 350 a 60 das, Federaltex S.A. de Valledupar s/fra. N 335 a 30 das por $
8.360.000; Letra girada por pagar a 60 das a nombre de Juan Sarasty & Ca. Ltda. de Bogot por $
23.260.000. Cmo ser el Informe del Inventario de Bienes y Obligaciones y cul es el Capital
Contable?
Solucin: se construye el Inventario de Bienes y Obligaciones del seor Johnny Piescalientes para
poder conocer su Patrimonio, as:

Para ver trabajos similares o recibir informacin semanal sobre nuevas publicaciones, visite www.monografias.com

www.monografias.com

6- Prepare el Balance de Apertura de la empresa Rosa de Sharon S.C.A. en 30 de Junio, si la seora


Rosa Mara Crespo, socia fundadora de la sociedad:
a) En 01/03 del mismo ao haba adquirido un edificio por $ 100.000.000, construido sobre un
terreno cuyo avalo haba sido estimado en el 15% del valor total
b) En 06/04 adquiri maquinaria por valor de $ 40.000.000 habiendo dado la mitad de contado y la
otra mitad en una letra a 30 das por el resto
c) En 09/05 compr una extensin de tierra por $ 40.000.000 en efectivo
d) En 14/06 vendi la extensin de tierra aceptando como pago $ 5.000.000 en efectivo, un camin
avaluado por $ 27.000.000 y equipo de oficina por $ 8.000.000

Para ver trabajos similares o recibir informacin semanal sobre nuevas publicaciones, visite www.monografias.com

www.monografias.com

e) En 30/06 organizo una SCA y emiti 5.000 acciones de $ 1.000.000 que compraron entre cinco
socios con la condicin de aceptar los bienes aportados por la Sra. Rosa Mara Crespo como
parte de pago.
Solucin: Mediante un Inventario de Bienes y Obligaciones de la seora Rosa Mara Crespo se
puede conocer el valor neto de sus aportes en especie as como el valor restante en efectivo para
completar la adquisicin requerida de acciones:

Siendo que a cada socio corresponden 1000 acciones con valor de $ 1.000.000 cada una, entonces
a la socia Rosa Mara Crespo aun le falta por aportar la suma de (1.000.000.000 155.000.000 = $
845.000.000) en efectivo. Entre los dems socios han de aportar la suma de $ 4.000.000.000 en
efectivo. En consecuencia, el Balance de Apertura, con el cual habran de abrirse los Libros de
Contabilidad de la nueva Sociedad, quedar as:

Para ver trabajos similares o recibir informacin semanal sobre nuevas publicaciones, visite www.monografias.com

www.monografias.com

7- Indique si cada uno de los trminos siguientes se identifica como Activo, Pasivo o Patrimonio
acegi-

Honorarios devengados
Materiales comprados
Sueldos del personal
Utilidades de Ejercicios
anteriores
Efectivo

A
A
P
Pn

bdfh-

j-

Cuentas por pagar


Terrenos
Reservas obligatorias
Cuentas Corrientes
Comerciales
Para obligaciones
fiscales

P
A
Pn
P
P

1.6.2. Problemas Propuestos


1- Cules son los tres tipos de actividades que se presentan en un Estado de Flujo de Efectivo?
Indique a que actividad del Estado de Flujo de Efectivo pertenece cada una de las siguientes
transacciones:
a) Se recibe efectivo como nueva inversin del propietario
b) Se paga en cheque por la compra de un terreno
c) Se recibe efectivo como honorarios por servicios prestados
d) Se paga en efectivo por gastos operacionales
2- Los recursos que posee un negocio se llaman:.
a- Gastos
b- Pasivos
c- Ingresos
d- Activos
e- Costos
3- Un negocio lucrativo cuya propiedad se divide en acciones se conoce como:
a- Empresa de servicios
b- Corporacin
c- Persona jurdica
d- Sociedad
e- Persona natural

Para ver trabajos similares o recibir informacin semanal sobre nuevas publicaciones, visite www.monografias.com

www.monografias.com

4- Por cuales de las siguientes transacciones comerciales no tendr el Contador que confeccionar
asientos contables?
a- La empresa vende mercanca
b- La empresa contrata a un auxiliar contable
c- La empresa asume el valor de la licencia de conduccin de uno de sus choferes
d- La empresa firma un contrato para constituirse en distribuidora exclusiva de los zapaticos
Chevrolet
e- La empresa paga una infraccin de trnsito cometida por la esposa del gerente
f- La empresa compra mercancas
5- Cules son los dos tipos de transacciones que aumentan el capital contable de una empresa
individual?. Explique.
6- La ecuacin patrimonial est conformada por la igualdad:
a) Pasivo + Activo = Patrimonio
b) Patrimonio + Activo = Pasivo
c) Activo Pasivo = Patrimonio
d) Activo + Patrimonio = Pasivo
e) Activo + Pasivo = Patrimonio
7- Si el total de Activos aumenta en $ 20.000.000 durante un periodo y el de Pasivos se incrementa en
$ 15.000.000, el cambio del Patrimonio durante el mismo periodo ha sido:
a- Aumentado en $35.000.000
b- Disminuido en 35.000.000
c- Aumentado en $ 50.000.000
d- Disminuido en $ 50.000.000
8- Describa el efecto de las siguientes transacciones de negocios sobre el Activo, el Pasivo o el
Patrimonio (A: Aumento o D: Disminucin, y A: Activo, P: Pasivo, o Pn : Patrimonio)
Efectivo invertido en el negocio
Retiros por parte del propietario
Pago por servicios pblicos

Efectivo recibido por servicios


prestados
Materiales comprados al contado
Materiales comprados a crdito

9- Si el total de Activos aumenta en $ 20.000.000 durante un periodo y el de Pasivos se incrementa en


$ 15.000.000, el cambio del Patrimonio durante el mismo periodo ha sido:
e- Aumentado en $35.000.000
f- Disminuido en 35.000.000
g- Aumentado en $ 50.000.000
h- Disminuido en $ 50.000.000
10- En las cuentas que se mencionan a continuacin:
Dinero en especie: $ 1.000.000
Carne de res comprada, aun sin recibir: $ 2.750.000
Una bicicleta de carreras. $ 20.000.000
Un vehculo todo terreno: $ 45.000.000
Deuda pendiente por el vehculo: $ 22.500.000
Recursos aportados por compaeros del negocio: $ 15.000.000
a) Identifique cual es el valor del patrimonio
b) Cul es el valor de los bienes,
c) Cul es el valor de los derechos
d) Cul es el valor de las obligaciones
11- Sabiendo que ACTIVOS = PASIVOS + CAPITAL CONTABLE, calcule la cantidad faltante en cada
una de las igualdades siguientes:
a)
= $ 20.500.000 + 51.700.000
b) 37.250.000 =

+ 12.300.000
c) 58.700.000 = 28.100.000 +

12- Construya el Inventario de Bienes y Obligaciones a 31 de Octubre de 2010 y determine el Capital


Contable del Centro Comercial Snchez Escolar, con base en la siguiente informacin:
APORTES AL NEGOCIO:
a) En Dinero, billetes y monedas en circulacin, $ 8.750.000
b) En Cheques, de Efectivo a la orden, $ 12.644.250

Para ver trabajos similares o recibir informacin semanal sobre nuevas publicaciones, visite www.monografias.com

www.monografias.com

13-

14-

1516-

c) En Depsitos a la orden en Cuentas Corrientes en Bancos, $ 44.515.300


d) En Mercancas en existencias, as:
- 100 sacos de caf excelso Manizales, de 50 Kg. c/u, comprados a $ 450.000 c/u. Precio de
Mercado: $ 430.000 c/u.
- 85 sacos de arroz Roa de 50 Kg. C/u, comprados a $ 70.000 c/u,
- 125 sacos de harina Insuperable de 67,5 Kg. C/u comprados a $ 80.000 c/u. 12 sacos se
hallan rodos por los ratones.
e) Una propiedad edificada de 540 metros cuadrados en el Barrio Pasatiempo de la ciudad de
Montera, a $ 600.000 el metro cuadrado, segn Escritura Pblica 2128 del 23 de Febrero de
2010 de la Notara 3 de la misma ciudad. El avalo de Camacol-Crdoba asigna al terreno el
valor de 40.000.000 de pesos.
f) Un pagar a cobrar el 10 de Noviembre de 2008 a cargo de Tatiana Padilla, de Sampus, por
valor de $ 2.750.000.
g) En diversas cuentas por cobrar por concepto de ventas a crdito, sin garanta especfica:
- Molinos La Rosa S.A. de Cartagena, para el 12/11/08: $ 24.420.000
- Torres y Torres Ltda. de Cali, para el 25/11/09: $ 33.683.170
- Hijos de M. Juliao Ltda. de Barranquilla, para el 8/12/2013: $ 99.750.320
DEUDAS A CARGO DEL NEGOCIO
h) En Letras aceptadas por pagar, as:
- Nmero 245 de Montoya e Hijos de Ibagu, a favor del Banco Agrario de Bogot, para el
25/11/12, por $ 1.200.000
- N 330 de Parker Lee Inc. de NY, a favor del Banco de Bogot para el 8/12/12 por US$ 1214
(tasa de cambio hoy: $ 2165/US$)
i) En Crditos concedidos por:
- Cox and Fox Ltd. de Los ngeles, por mercancas compradas a ellos, sin garanta especfica,
por valor de US$ 932
- Mavesa, de Venezuela, por la misma razn anterior, por valor de Bs. 500.000 (tasa de cambio
hoy: $ 3,29 /Bs.)
j) En Hipoteca a favor del Banco Ganadero de Montera, por Escritura Pblica N 2129 del
23/02/10 de la Notara 3 de la misma ciudad con vencimiento el 18/10/13 por la suma de $
60.000.000.
Una sociedad ha sido autorizada para que comience su negocio con una emisin de acciones
mxima de 10.000 acciones, cada una con un valor de $ 500.000.
a) Si el Sr Gutirrez compra de contado 2.000 acciones, cuanto paga?
b) Si el Sr lvarez adquiere 500 acciones a cambio de un billete de lotera que el prximo 31 de
Diciembre juega 5 billones de pesos. El Sr lvarez haba pagado por el billete $ 1.000.000 por el
billete. Cuantas acciones cree Ud. que vale el billete de lotera?. Explique.
Prepare el Balance de Apertura de la empresa Ferretera de Pedro Quijano Rueda que antes de
constituirse en sociedad, su fundador, haba realizado las siguientes transacciones:
a) En 06/01 compr maquinaria al contado por $ 125.000.000 y equipo de oficina a crdito por $
20.000.000 con un plazo de 25 das para su pago.
b) En 06/02 compr un edificio habiendo entregado $ 200.000.000 de contado y una letra a 90 das
por $ 700.000000. Se calcula que el valor del terreno es de $ 150.000.000
c) En 08/02 se organizo la sociedad PQR SAS y emiti un total de 2.000 acciones a $ 1.000.000
cada una, cuya venta fue hecha al contado, admitiendo como parte de pago, los bienes
aportados por el socio fundador.
SUGERENCIA: Primero represente en cuentas T las transacciones del fundador incluyendo la
transformacin en sociedad y luego, confecciones el Balance de Apertura en un Comprobante
de Diario.
Por medio de una cuenta T, calcule el saldo dbito o crdito de la cuenta
a) COSTO DE MERCANCAS, cuyo movimiento dbito es: $ 320.000, 250.000, y 125.000
b) INGRESOS FINANCIEROS, cuyo movimiento crdito es $ 490.000, 390.000 y 190.000
Explique cul es el efecto sobre el Activo, el Pasivo y el Patrimonio, de cada una de las siguientes
transacciones
a) Un terreno comprado por $ 100.000.000 de los cuales se quedan debiendo $ 20.000.000
b) El terreno se vende al contado por $ 180.000.000
c) Despus de recibir los $ 180.000.000 en efectivo, el vendedor paga los $ 20.000.000 que
adeudaba.

Para ver trabajos similares o recibir informacin semanal sobre nuevas publicaciones, visite www.monografias.com

www.monografias.com

17- Indique si cada uno de los siguientes tipos de transaccin incrementa o disminuye el capital
contable:
a)
b)
c)
d)

Nuevas inversiones del propietario en el negocio


Mas ingresos
Gastos no operacionales
Retiros por parte del propietario

18- Un escritorio que cuesta $ 1.000.000 antes de temporada escolar se vende por $ 600.000 despus
de esta. Si una compaa fuera a comprar este escritorio, depender el valor que le vaya a asignar
al activo del momento en que haga la compra, es decir, de si antes o despus de la temporada
escolar?. Explique.
19- En el siguiente cuadro de resumen del Balance General y del Estado de Resultados de cuatro
empresas, aparecen algunos rubros faltantes. Srvase llenarlos con la respuesta correcta:
Al principio del ao
Activos
Pasivo
Capital Contable
Durante el ao
Inversin adicional
Retiros del propietario
Ingresos
Gastos
Utilidad
Al final del ao
Activos
Pasivo
Capital Contable

I
950.000.000
490.000.000

II

180.000.000

III
900.000.000
720.000.000

IV
500.000.000
380.000.000

220.000.000
80.000.000

120.000.000

50.000.000
75.000.000
150.000.000
140.000.000

500.000.000

90.800.000
80.900.000

48.000.000
200.000.000
142.000.000

150.000.000
47.000.000

350.000.000
190.000.000

950.000.000
820.000.000

900.000.000
530.000.000

20- El seor Antonio Insignares se propone abrir un negocio de venta de artculos para el hogar el 30 de
Agosto y empieza aportando una inversin de $ 150.000 en efectivo. Le pone como nombre el
Buen Marido. Elabore un Inventario de Bienes y Obligaciones el da en que el negocio vaya a
comenzar las operaciones y confeccione en un Comprobante de Diario su Balance de Apertura:
a) En septiembre 2, compra muebles y enseres para el negocio por $ 900.000, cancelando $
300.000 en efectivo y queda debiendo el resto
b) En septiembre 7 compra mercancas por $ 20.000.000, cancela en efectivo $ 3.500.000 y para
el resto firma 5 letras de cambio
c) En septiembre 12 compra un edificio por $ 190.000.000, cancela $ 10.000.000 en efectivo y por
el resto firma una hipoteca con el banco social. El valor del terreno es de $ 30.000.000
d) En septiembre 17 aparta $ 300.000 para tramitar la licencia de funcionamiento del negocio en la
oficina de impuestos distritales.
e) En septiembre 22 compra papelera y tiles por $ 70.000 en efectivo
f) En septiembre 27 compra equipo de oficina por $ 600.000, cancelando $ 200.000 en efectivo y
el resto lo queda debiendo
g) Finalmente, el negocio quedo listo para empezar a funcionar el da 1 de Octubre.
Las cuentas
2.1. Concepto de Cuenta
Cuentas es la denominacin que la Contabilidad ha dado al lugar o parte imaginaria donde se acumulan los
valores que se reciben (DEBE) y los que se entregan (HABER). La cuenta es un instrumento de
representacin y medida de un elemento patrimonial que se utiliza para captar la situacin inicial de ste y
las variaciones que posteriormente se vayan produciendo en el mismo (LOPES DE SA, 2009).

Para ver trabajos similares o recibir informacin semanal sobre nuevas publicaciones, visite www.monografias.com

www.monografias.com

La cuenta es el componente ms elemental y bsico de la Contabilidad. Es como una ficha de anotacin


individual donde se anotan todos los movimientos econmicos (contables) como sucede, por ejemplo, en las
cuentas de: CAJA, CLIENTES, BANCOS, etc.
Al dar un nombre a una cuenta, ste debe ser tan claro, explcito y completo que por el solo nombre se
identifique lo que representa. Las cuentas representan las propiedades, derechos y deudas de una empresa
en una fecha determinada; as mismo, los costos, los gastos y las utilidades en un perodo determinado.
De acuerdo con su naturaleza, las cuentas se clasifican en reales o de balance, nominales o resultado y,
cuentas de orden.
CLASIFICACIN DE LAS CUENTAS
Las cuentas se clasifican en reales o de balance, nominales o de ganancias y prdidas, y de orden. El
siguiente diagrama ilustra la clasificacin de las cuentas:

Cuentas permanentes, reales o de balance: Las cuentas reales representan valores tangibles como las
propiedades y las deudas; son temporales, porque forman parte del balance general de la empresa. Las
cuentas reales, a su vez, pueden ser de Activo, Pasivo o Patrimonio.
Activo:
Representa todos los bienes y derechos apreciables en dinero, de propiedad de la empresa. Se entienden
por bienes, entre otros, el dinero en caja o en bancos, las mercancas, los muebles y los vehculos; y por
derechos, las cuentas por cobrar y todos los crditos a su favor (CORAL y GUDIO, 2008).
Caractersticas: Entre las principales caractersticas de los activos, tenemos
Estar en capacidad de generar beneficios o servicios.
Estar bajo el control de la empresa.
Generar un derecho de reclamacin.
Movimiento de las Cuentas de Activo: estas cuentas empiezan y aumentan su movimiento en el Debe;
disminuyen y se cancelan en el Haber, por lo general su saldo es debito.
Conforman la clase de cuentas de Activo: Caja, Bancos, Clientes, Acciones, Terrenos y otras que para su
conocimiento y manejo encontrar usted, en forma detallada, en el Plan nico de Cuentas, un poco ms
adelante.
Pasivo:
Representa todas las obligaciones contradas por la empresa para su cancelacin en el futuro. Son las
deudas que debe pagar por cualquier concepto. (CORAL y GUDIO, 2008)
Caractersticas: Entre las principales caractersticas de los Pasivos, tenemos:
La obligacin tiene que haberse causado, lo cual implica el deber de hacer el pago
Existe una deuda con un beneficiario cierto.
Movimiento de las Cuentas de Pasivo: estas cuentas empiezan y aumentan su movimiento en el Haber;
disminuyen y se cancelan en el Debe, por lo general su saldo es crdito.
A la clase de cuentas de Pasivo pertenecen: Obligaciones con Bancos Nacionales, Proveedores Nacionales,
Salarios por Pagar, y otras que usted aprender a manejar con el Plan nico de Cuentas.
Patrimonio:
Matemticamente, el Patrimonio se calcula por la diferencia entre el Activo y el Pasivo. Representa los
aportes del dueo o dueos para constituir la empresa y adems incluye las utilidades y reservas. Este
grupo constituye un Pasivo a favor de los dueos de la empresa, por lo cual se maneja de igual forma que
las cuentas de Pasivo. (CORAL y GUDIO, 2008)

Para ver trabajos similares o recibir informacin semanal sobre nuevas publicaciones, visite www.monografias.com

www.monografias.com

Caractersticas: Entre las principales caractersticas del Patrimonio, tenemos:


- Est constituido por el aporte inicial y el aporte adicional de los dueos de la empresa.
- Se incrementa con las utilidades y se disminuye con las prdidas del ejercicio contable.
- Movimiento de las Cuentas de Patrimonio: su movimiento es igual que las cuentas del Pasivo.
Cuentas Temporales, Nominales o de Resultado: Las cuentas nominales son llamadas tambin de resultado
porque al final de un ejercicio contable dan a conocer las utilidades, costos y gastos; y son temporales
porque han de ser canceladas al cierre del perodo contable. Comprende las cuentas del Estado de
Ganancias y Prdidas: de Ingresos, Gastos Costo de Ventas.
Ingresos:
Son los valores que recibe la empresa cuando realiza una venta o presta un servicio con el fin de obtener
una ganancia (CORAL y GUDIO, 2008).
Movimiento de las cuentas de Ingresos: estas cuentas empiezan y aumentan en el Haber; normalmente no
se debitan sino al final del ejercicio para cancelar su saldo.
Gastos:
Una empresa debe efectuar pagos por los servicios y elementos necesarios para su buen funcionamiento;
esto constituye un gasto o una prdida porque esos valores no son recuperables. Ejemplos: el pago de
sueldos, arrendamientos, servicios pblicos y otros. (CORAL y GUDIO, 2008)
Movimiento de las cuentas de Gastos: estas cuentas empiezan y aumentan en el Debe; disminuyen y se
cancelan en el Haber. Su saldo es dbito.
Costo de Ventas:
Representa el valor de adquisicin de los artculos destinados a la venta. (CORAL y GUDIO, 2008)
Movimiento de las cuentas de Costo de Ventas: estas cuentas empiezan y aumentan en el Debe,
disminuyen y se cancelan en el Haber; normalmente no se acreditan; su saldo es dbito.
Cuentas de Orden: Las cuentas de orden reflejan hechos o circunstancias contingentes que pueden llegar a
afectar la estructura financiera de la empresa. En ellas se incluyen tambin aquellos contratos sobre futuras
situaciones operacionales y dems cuentas de registro para efectos de control interno o de informacin
gerencial conveniente.
Cuentas de orden deudoras:
Los dineros, documentos y bienes que son de propiedad de la empresa pero que estn en poder de
terceros, conforman el grupo de cuentas de orden deudoras y se manejan como las cuentas de Activo, por
lo tanto su saldo es dbito. Un ejemplo es la cuenta llamada: Bienes y Valores Entregados en Custodia.
(CORAL y GUDIO, 2008)
Cuentas de orden acreedoras:
Los bienes que se han recibido para custodia, manejo o negociacin sin ser de nuestra propiedad,
constituyen el grupo de cuentas de orden acreedoras, stas se manejan como cuentas de Pasivo, por lo
cual su saldo es crdito. Ejemplo: Acreedores, Bienes y Valores Recibidos en Custodia (CORAL y GUDIO,
2008).
Es importante destacar que ambos grupos de cuentas de orden no afectan ni el Activo, ni el Pasivo, ni el
Patrimonio, sino que representan un rol contingente, es decir, que eventualmente, podran o no, tener
efectos sobre nosotros..
2.2. Naturaleza dbito o crdito de un registro
En otras palabras Cmo sabemos si la cantidad la tenemos que colocar en el DEBE o en el HABER?, Por
ejemplo en la cuenta de CAJA por qu el asiento 1 es un debito y porque el asiento 2 es un crdito, porque
se hizo as? Todo depender del tipo de cuenta que sea.
Slo debemos conocer la naturaleza de la cuenta para saber si debemos asentar la cantidad en el DEBE o
en el HABER. Dentro de las cuentas que hacen parte del Balance General (las cuentas Reales o
Permanentes), el Activo de naturaleza debito, mientras que las cuentas del Pasivo y del Patrimonio son de
naturaleza crdito. Entre las cuentas que forman parte del Estado de Resultados (las cuentas nominales o
Temporales), el Gasto y el Costo son de naturaleza debito mientras que la del Ingreso es de naturaleza
crdito. Obsrvese el siguiente esquema:

Para ver trabajos similares o recibir informacin semanal sobre nuevas publicaciones, visite www.monografias.com

www.monografias.com

PROCEDIMIENTO PARA REGISTRAR LAS OPERACIONES MERCANTILES:


El procedimiento para registrar correctamente una operacin mercantil es el siguiente:
1. Identificar cual es el tipo de transaccin en el que se puede ubicar el asiento contable que se quiere
registrar.
Para ello el interrogante que nos hacemos es cul es la clase de hecho econmico que vamos a
convertir en un hecho contable. Es una compra o una venta?. Es un bien o una mercanca?. Es a
crdito o al contado?. Se paga en efectivo o en cheque?. Debemos recordar que anteriormente
hemos establecido la existencia de 17 tipos particulares de transacciones econmicas.
2. Determinar la cuenta que se debita y la cuenta que se acredita.
Para ello nos debemos preguntar si se trata de una o ms cuentas de Activos, una o ms cuentas
de Pasivos, uno o ms cuentas de Patrimonio, una o ms cuentas de Gastos, una o ms cuentas de
Costos, o una o ms cuentas de Ingresos. Siempre se ha de tener en cuenta que

3. Comprobar la igualdad de la suma de los dbitos y los crditos


Para ello nos debemos preguntar si se trata de uno o ms asientos simples o compuestos
EJERCICIO DE APLICACIN
1- El Almacn Pintura Arte y Decoracin, el Sbado 15 de Agosto, vendi mercancas al
contado por valor de $ 6.300.000
Para estudiar el registro de las transacciones pasamos nuevamente al esquema del libro diario o bitcora
contable:
Por ejemplo, la cuenta CAJA es una cuenta de ACTIVO, as que si ingresamos dinero, debemos colocarlo
en el DEBE, y si sacamos, en el HABER.
Fecha
15/08/20xx

Cdigo

Nombre de la Cuenta
Caja
Comercio al P/M y al p/m

Debe
6.300.000

Haber
6.300.000

2- El da Lunes a primera hora consigna en el Banco de Occidente la suma de $ 6.000.000


Segn el esquema anterior, BANCOS es una cuenta de ACTIVO y su aumento se reflejara segn el grfico
anterior.
Fecha
17/08/20xx

Cdigo

Nombre de la Cuenta
Bancos

Debe
6.000.000

Haber

Para ver trabajos similares o recibir informacin semanal sobre nuevas publicaciones, visite www.monografias.com

www.monografias.com

Caja

6.000.000

3- El da Martes gira un cheque por $ 300.000 a nombre de Ferretera Cordilleras Ltda. por
concepto de abono a deuda por mercancas
Fecha
18/08/20xx

Cdigo

Nombre de la Cuenta
Proveedores
Bancos

Debe
300.000

Haber
300.000

4- El da Mircoles compra a Epxicas y Acrlicas S:A: mercancas al contado por $ 500.000


que cancela con cheque de Banoccidente
Fecha
19/08/20xx

Cdigo

Nombre de la Cuenta
De Mercancas
Bancos

Debe
500.000

Haber
500.000

5- El da Jueves compra mercancas a crdito a Terin S.A. por $ 400.000


Fecha
20/08/20xx

Cdigo

Nombre de la Cuenta
De Mercancas
Proveedores

Debe
400.000

Haber
400.000

6- El da Viernes vende mercancas a crdito al Colegio Marymount por $ 1.700.000


Fecha
21/08/20xx

Cdigo

Nombre de la Cuenta
Clientes
Comercio al P/M y al p/m

Debe
1.700.000

Haber
1.700.000

7- La empresa obtiene un prstamo ordinario de Banoccidente por $ 3.000.000 que consigna


directamente en la cuenta corriente
Fecha
21/08/20xx

Cdigo

Nombre de la Cuenta
Bancos
Bancos Nacionales (pagares)

Debe
3.000.000

Haber
3.000.000

8- El colegio Marymount le abona $ 250.000 a la deuda


Fecha
21/08/20xx

Cdigo

Nombre de la Cuenta
Caja
Clientes

Debe
250.000

Haber
250.000

9- El Lunes 24 la empresa compra Mercancas a crdito a Panamericana S.A. por $ $ 1.050.000


Fecha
24/08/20xx

Cdigo

Nombre de la Cuenta
De Mercancas
Proveedores

Debe
400.000

Haber
400.000

10- El Martes la empresa Barrio Montes EU anuncia que para seguir atendiendo los pedidos de
los clientes debern mantener un fondo permanente equivalente al 10% de su pedido
promedio, por lo que la empresa le hace un giro anticipado de $ 350.000
Fecha
25/08/20xx

Cdigo

Nombre de la Cuenta
Cuentas corrientes comerciales
Bancos

Debe
350.000

Haber
350.000

Para ver trabajos similares o recibir informacin semanal sobre nuevas publicaciones, visite www.monografias.com

www.monografias.com

11- El Mircoles gira un cheque por $ 1.000.000 al Banoccidente como abono a su deuda por el
prstamo ordinario recibido
Fecha
26/08/20xx

Cdigo

Nombre de la Cuenta
Bancos Nacionales (pagares)
Bancos

Debe
1.000.000

Haber
1.000.000

12- El Jueves llegan los recibos de las cuentas de agua y telfonos a cargo del dueo del
negocio por $ 340.000, sin embargo le es solicitado a Parmnides Sarmiento, hacer el pago
mientras se perfecciona el contrato de unos negocios que se estn definiendo.
Fecha
31/08/20xx

Cdigo

Nombre de la Cuenta
Servicios
Cuentas corrientes comerciales

Debe
340.000

Haber
340.000

13- El Viernes recibe la propuesta de Mario Bross para ser socio, aportando la suma de $
1.500.000, con cheque y tambin, Parmnides Sarmiento quien aporta un punto de venta
cuya prima es avaluada por $ 2.000.000. Estos hechos dan lugar a la transformacin de la
Empresa de Unipersonal a Sociedad Limitada.
Fecha
27/08/20xx

Cdigo

Nombre de la Cuenta
Caja
Crdito Mercantil
Capital de Personas Naturales
Aportes Sociales

Debe
1.500.000
2.000.000
214.805.250

Haber

218.305.250

14- La empresa gira un cheque por $ 360.000 como pago de un mes de arrendamiento del local
Fecha
28/08/20xx

Cdigo

Nombre de la Cuenta
Arrendamiento
Bancos

Debe
360.000

Haber
360.000

15- La empresa gira otro cheque por $ 480.000 a la empresa Vigilancia del Norte por los meses
anticipados de Septiembre y Octubre de servicios de seguridad nocturna.
Fecha
28/08/20xx

Cdigo

Nombre de la Cuenta
Gastos pagados por anticipado
Bancos

Debe
480.000

Haber
480.000

16- La empresa recibe $ 4.200.000 por concepto de utilidad en venta de equipo de minas y
canteras que se encontraba abandonado y totalmente depreciado.
Fecha
29/08/20xx

Cdigo

Nombre de la Cuenta
Caja
Utilidad en venta de propiedades

Debe
4.200.000

Haber
4.200.000

17- La cadena de Ferreteras Kike anticipa la suma de $ 900.000 para que le garanticen la
atencin por un nuevo contrato
Fecha
31/08/20xx

Cdigo

Nombre de la Cuenta
Caja
Ingresos recibidos por anticipado

Debe
900.000

Haber
900.000

QU SE HACE CON LOS ASIENTOS DE DIARIO?

Para ver trabajos similares o recibir informacin semanal sobre nuevas publicaciones, visite www.monografias.com

www.monografias.com

Se debe comparar la igualdad de las sumas de los dbitos y de los crditos, siguiendo el orden numrico de
los cdigos de las cuentas, utilizando para ello el instrumento denominado cuentas T, as:

Los movimientos correspondientes se resumen para pasarlos al formato denominado Comprobante de


Diario, antes de ser registradas en el Libro Mayor, as:

Para ver trabajos similares o recibir informacin semanal sobre nuevas publicaciones, visite www.monografias.com

www.monografias.com

Para ver trabajos similares o recibir informacin semanal sobre nuevas publicaciones, visite www.monografias.com

www.monografias.com

Es muy importante mencionar aqu, que las cuentas T, como un formato didctico, tiene dos utilizaciones, la
primera, que ya hemos estado trabajando, se interesa en la determinacin de los movimientos (dbito y
crdito) para ser utilizados en la confeccin del Resumen o Comprobante Diario y, la segunda, que
representa lo que ocurre en el Libro Mayor, se interesa en la determinacin del saldo de la cuenta arrojando
los mismos valores obtenidos en la confeccin de un Balance de Comprobacin de saldos, tal como
veremos ms adelante.
2.3. Dinmica de los Hechos Contables
CLASIFICACION DE LOS HECHOS CONTABLES
Segn las modificaciones que se produzcan en el activo, en el pasivo, el patrimonio neto o en los
resultados, podemos clasificar los hechos contables en:
Permutativos:
Son los hechos que se pueden alterar alguno de los elementos de la ecuacin patrimonial (activo, pasivo o
patrimonio) pero sin ocasionar prdidas ni ganancias. Por ejemplo el ingreso de efectivo en el banco, o la
compra de un vehculo al contado. Se cambia un elemento de activo que sube por otro que baja.
Aumenta un activo, disminuye un activo: Ejemplo: Deposito efectivo en el banco
Fecha
17/08/20xx

Cdigo

Nombre de la Cuenta
Bancos
Caja

Debe
6.000.000

Haber
6.000.000

Aumento de un pasivo, disminucin de un pasivo: Ejemplo: Documento la deuda que tena con el dueo del
local
Fecha
31/08/20xx

Cdigo

Nombre de la Cuenta
Cuentas Corrientes Comerciales
Costos y Gastos por pagar

Debe
340.000

Haber
340.000

Disminuye un pasivo, disminuye un activo: Ejemplo: Pago en efectivo un documento por pagar

Para ver trabajos similares o recibir informacin semanal sobre nuevas publicaciones, visite www.monografias.com

www.monografias.com

Fecha
18/08/20xx

Cdigo

Nombre de la Cuenta
Proveedores
Bancos

Debe
300.000

Haber
300.000

Modificativos:
Son hechos que alteran los resultados, bien aumentando o bien disminuyendo el activo o el pasivo. Por
ejemplo, el pago de una reparacin al contado, o el abono de intereses de una cuenta corriente.
Aumento de un activo, aumento de un ingreso: Ejemplo: Cobro parcial del nuevo contrato
Fecha
31/08/20xx

Cdigo

Nombre de la Cuenta
Caja
Ingresos recibidos por anticipado

Debe
900.000

Haber
900.000

Aumento de un gasto disminucin de un activo: Ejemplo: Pago intereses por mora


Fecha
28/08/20xx

Cdigo

Nombre de la Cuenta
Gastos pagados por anticipado
Bancos

Debe
900.000

Haber
900.000

Mixtos:
Son hechos que son a la vez permutativos y modificativos. Por ejemplo la venta de un vehculo usado por
mayor o menor valor al de su valor en libros.
Aumento de un activo, aumento de patrimonio: Ejemplo: Cuando recibes un activo como aporte (digamos,
cierta cantidad de dinero y la prima del local comercial)
Fecha
31/08/20xx

Cdigo

Nombre de la Cuenta
Caja
Crdito Mercantil
Aportes sociales

Debe
1.500.000
2.000.000

Haber

3.500.000

Aumento de un gasto, aumento de pasivos y de patrimonio: Ejemplo: Cuando se crea la provisin del
impuesto a la renta (33%) y de las reservas obligatorias (10%)
Fecha
31/08/20xx

Cdigo

Nombre de la Cuenta
Ganancias y Perdidas
Para Obligaciones Fiscales
Reservas Obligatorias
Utilidades del Ejercicio

Debe
100.000.000

Haber
33.000.000
10.000.000
57.000.000

LOS LIBROS DE CONTABILIDAD


Una vez definido lo que son hechos contables, el prximo paso consiste en indicar la forma de registrar la
historia de estas transacciones, lo cual consiste en llevar libros para estos fines.
En materia de los libros que deben llevar los comerciantes la Superintendencia de Sociedades se ha
pronunciado reiteradamente manifestando que al no haber fijado la ley de manera expresa cuales son, estos
deben sujetarse a los lineamientos establecidos en el art. 125 del Decreto 2649 de 1993, en concordancia
con los arts. 50, 52 y 53 del Estatuto Mercantil o Cdigo de Comercio, de donde se desprende que para que
la contabilidad sea comprensible y til y las operaciones sean registradas en estricto orden cronolgico, bien
sea de manera individual o por resmenes globales no superiores a un mes, es necesaria la utilizacin de
los llamados libros Diario y Mayor.
Los libros de contabilidad pueden ser comprados o pueden ser mandados a hacer. Pueden ser de hojas
removibles o formarse por un consecutivo de tarjetas. Tambin pueden ser guardados en medios
magnticos, pero en muchos pases estas formas tecnolgicas aun no constituyen medios de prueba judicial
en cuestiones mercantiles. En todo caso, sus formas impresas deben estar clasificadas, encuadernadas,
forradas, foliadas, rubricadas y archivadas en orden, para garantizar su integridad y autenticidad.

Para ver trabajos similares o recibir informacin semanal sobre nuevas publicaciones, visite www.monografias.com

www.monografias.com

Son presentadas a la Cmara de Comercio y a la Administracin de Impuestos Nacionales, quienes


registran en la primera hoja los requisitos que caracterizan a cada clase de libro e imprimiendo el sello y
rubricando todas las dems paginas respectivas.
El Libro Diario:
En el Libro Diario se anotan las transacciones diarias en orden cronolgico, esto es, se anotan todos los
actos de comercio de tal forma "como si fuera nuestro diario personal donde anotamos los acontecimiento
de nuestra vida diaria o la bitcora de un barco, que registra la historia comentada de cada viaje".
De la misma manera, el libro diario comercial es el libro del registro original y est destinado a recoger por
orden cronolgico y da por da las operaciones mercantiles que ejecute el empresario y tambin todas las
que puedan influir de algn modo en la situacin financiera, expresando detalladamente el carcter, las
circunstancias y los resultados de cada uno de ellos.
En general, el Diario consta de: a) columna para fechas, b) espacio para los nombres de las cuentas
deudoras y acreedora, debajo de las cuales se explica en forma clara y precisa la operacin que las motiva,
c) dos columnas para anotar en la de la izquierda los valores asumidos por las cuentas deudoras y en la de
la derecha los de las cuentas acreedoras.
Los negocios que efectan un nmero grande de transacciones por da, o aquellos negocios que han
alcanzado un mayor nivel de organizacin en su contabilidad, emplean un Diario Multicolumnario, el cual
contiene de 10 a 30 columnas. En estos Diarios Multicolumnarios se anotan diariamente los Resmenes o
Comprobantes de Diario de cada da y cada vez que finalice el registro de las operaciones de un mes, se
totaliza el movimiento debito y crdito de cada cuenta y, sin incluir los resmenes de apertura y de meses
anteriores, estos valores se trasladan a las columnas del movimiento mensual correspondiente en el Libro
Mayor (CORAL y GUDIO, 2008)
La manera como cada empresa distribuye las columnas en su Diario Multicolumnario depende del
movimiento de las diferentes cuentas. Aquellas cuentas que con ms frecuencia se debitan y acreditan, se
les asigna dos columnas, tales como a Caja, Clientes, Proveedores, etc., en cambio, las cuentas que pocas
veces se debitan tales como compras De Mercancas o cualquier otra cuenta de gastos, tendrn una sola
columna (debito) y en igual forma, aquellas que pocas veces se acrediten, tales como Comercio al por
Mayor y al por menor u otras cuentas de ingreso, llevara tambin una sola columna (crdito). Al final, habr
una columna general para registrar los dbitos o los crditos de aquellas cuentas que no tienen columnas
especiales.
El Libro Mayor:
El Libro mayor se inicia con los saldos de apertura y se contina con los registros mensuales de los saldos
en resumen cuenta por cuenta, de los movimientos de un mes realizados y anotados en el diario. Esta
informacin es la base para realizar los estados financieros.
Cada final de mes, una vez totalizados el movimiento debito y crdito de cada cuenta y elaborado el
Resumen o Comprobante Diario del mes correspondiente se procede a su traslado al libro mayor. Para
confirmar la correccin de los pases al mayor, la suma de las anotaciones deudoras ha de ser igual a la
suma de las anotaciones acreedoras.
El rayado del Mayor, en su forma corriente, solo difiere del Diario en que tiene una columna de mas para
anotar los saldos que se van formando luego de cada anotacin debito o crdito. Algunos movimientos de
singular importancia, tales como Bancos, Clientes, Proveedores, etc., suelen dividirse en Mayores Auxiliares
distribuidos en orden alfabtico o por sucursales, o geogrficamente, segn convenga. Estos auxiliares
estn controlados por la cuenta del Mayor General en la que se anotan los totales resumidos, que han sido
anotados en los auxiliares con ms detalles. De esta manera, el saldo del Mayor General debe ser igual a la
suma de los saldos de todos los auxiliares.
El Libro de Inventarios y Balances:
Este libro est destinado para la anotacin de los inventarios de Bienes y Obligaciones al iniciar las
operaciones del negocio, as como para los Balances de cada final de periodo. El Balance es un cuadro
resumido y sinttico, que presenta los saldos de las cuentas de Activo, Pasivo y Patrimonio, cumpliendo con
ello la ecuacin patrimonial. Su rayado consta de columnas con espacios suficientes para anotar la
cantidad, calidad, nombre, precios unitarios, valor parcial y valor total
Teniendo en cuenta los requisitos legales que sean aplicables segn el tipo de acto de que trate, los hechos
econmicos deben documentarse mediante soportes, de orgenes internos o externos, debidamente
fechados y autorizados por quienes intervengan en ellos o los elaboren. Los soportes deben adherirse a los
comprobantes de contabilidad respectivos o, dejando constancia en estos de tal circunstancia (Art. 123 del
Decreto 2649 de 1993). Los estados financieros deben elaborarse con fundamento en los libros en los
cuales se hubieren asentado los comprobantes (Art. 125 del Decreto 2649 de 1993).

Para ver trabajos similares o recibir informacin semanal sobre nuevas publicaciones, visite www.monografias.com

www.monografias.com

Los entes econmicos deben conservar debidamente los libros de contabilidad, de actas, de registro de
aportes, los soportes de los asientos de contabilidad, los comprobantes de las cuentas y la correspondencia
relacionada con sus operaciones (CORAL y GUDIO, 2008). Los documentos que deben conservarse
pueden destruirse despus de veinte aos contados desde el cierre de los negocios. Estos podrn
destruirse transcurridos los diez ao siempre y cuando sea garantizada su reproduccin por cualquier otro
medio.
El Balance de Comprobacin
Se trata de un documento contable elaborado voluntariamente por el empresario que le permite hacerse una
idea sobre el desempeo de su empresa, con base en el saldo de cada una de las cuentas (ya sea deudor o
acreedor) arrojado por las operaciones del ejercicio contable,
El Balance de Comprobacin de Saldos de las cuentas por filas y de las Sumas por columnas, no requiere
de una periodizacin contable, ni de tener que establecer cul fue el resultado del periodo, ni mucho menos
requiere hablar de cierre de la contabilidad del periodo, para su elaboracin.
Es posible que el balance de comprobacin de sumas y saldos sea correcto y que, sin embargo, incluya una
contabilidad defectuosa. Este sera el caso, por ejemplo, de alguien que ha pagado a un proveedor pero que
en su registro haya anotado dicho pago a otro. El balance sera correcto numricamente pero no
contablemente.
Un Balance de Comprobacin apenas permite comprobar que la contabilidad de la empresa est bien
organizada y, ms bien, ser la revisin posterior de las pginas del Mayor, una por una, en comparacin
con el balance de Sumas y Saldos, lo que nos d la garanta de que todo est contablemente en orden.
Para su elaboracin se siguen los siguientes pasos:
1- Se escribe en el encabezamiento: el nombre de la empresa, el nombre del documento y la fecha de
elaboracin, en 3 lneas, centradas.
2- Se escriben los encabezamientos de las columnas, a saber: en las dos primeras columnas Balance
Inicial (o del periodo anterior), en las siguientes dos columnas Movimientos (del ejercicio) y en las
dos columnas finales Balance Final (o actual).
3- Los movimientos registrados por el Comprobante de Diario, del ejercicio de operaciones bajo
consideracin, se anotan horizontalmente en la lnea correspondiente de cada cuenta, en estricto
orden numrico indicado por el cdigo de cuentas. De ser necesario, se insertaran nuevas lneas
para las cuentas que no haban sido relacionadas, teniendo que desplazar las cuentas del Balance
del Periodo anterior ya anotado en las dos primeras columnas.
4- Se obtiene el Balance Final o actual, sumando horizontalmente los movimientos correspondientes
de cada cuenta incluidos las anotaciones del Periodo anterior ms la del ejercicio en consideracin.
Para obtener el resultado horizontalmente en cada cuenta de Activos o de Gastos, se suman
dbitos ms dbitos y se restan los crditos. En caso de ser cuentas de Pasivos o de Ingresos, se
suman crditos ms crditos y se restan los dbitos.
5- Se suman las columnas Debe y Haber resultantes, las cuales deben arrojar sumas iguales.

Para ver trabajos similares o recibir informacin semanal sobre nuevas publicaciones, visite www.monografias.com

www.monografias.com

Lo cual en cuentas T (representando al Libro Mayor), ser:

Para ver trabajos similares o recibir informacin semanal sobre nuevas publicaciones, visite www.monografias.com

www.monografias.com

2.6. Evaluacin
2.6.1. Problemas Resueltos
1- Frente a cada una de las cuentas que se relacionan a continuacin, escriba una A si es de
Activo, una P si es de pasivo, una K si es de patrimonio, una I si es de Ingreso, una G si es de
Gasto y una C si es de Costo

Para ver trabajos similares o recibir informacin semanal sobre nuevas publicaciones, visite www.monografias.com

www.monografias.com

Caja General
Ventas
Proveedores Nacionales
Terrenos Urbanos
Servicios pblicos
Gastos pagados por anticipado
Impuesto sobre ventas por pagar
Compras
Clientes nacionales
Ingresos recibidos por anticipado

A
I
P
A
G
A
P
C
A
P

Capital suscrito y pagado


Edificios
Retencin en la fuente
Costo de Ventas
Depreciacin acumulada
Crdito mercantil
Devoluciones en compras
Equipo de procesamiento de datos
Reservas obligatorias
Depreciaciones

Pn
A
P
C
A
A
C
A
Pn
G

2- Confeccione los asientos de diario de las siguientes transacciones de la empresa HIJ S.A. y
prepare un Resumen o Comprobante de Diario
a) En noviembre 1 emite acciones por la suma de $ 800.000.000, cuyo pago fue recolectado
as: $ 600.000.000 en efectivo y $ 200.000.000 en maquinarias.
b) Ese mismo da consigna en el Banco Davivienda el efectivo recolectado.
c) En noviembre 2 compr equipo de oficina a Danaranjo, a crdito por 60 das, al valor de $
15.000.000
d) En noviembre 3 hizo un desembolso de $ 1.000.000 para mantenimiento y reparacin
e) En Noviembre 9 recibi una factura por $ 110,000,000 de la empresa XYZ por la recepcin
de pedidos del almacn
f) En noviembre 10 envi una factura por $ 15.000.000 a la empresa OPQ por mercancas
despachadas del almacn
g) En Noviembre 19, pag salarios por $ 6.000.000
h) En Noviembre 26, carg a la deuda con la empresa XYZ la suma de $ 90.000.000
i) En Noviembre 29, obtuvo del Banco popular un prstamo de $ 40.000.000 respaldado por
un pagar
j) En Noviembre 30. pag arrendamiento del mes de Noviembre por $ 2.000.000
k) En Noviembre 30 pago sueldos por $ 8.000.000
l) En Noviembre 30, pago $ 15.000.000 con cargo a Danaranjo.
Solucin: Tras el paso intermedio de elaboracin del Resumen o Comprobante de Diario, se
hacen las anotaciones en el Libro Mayor:
a) Esta primera transaccin de Aportes de Capital dar lugar al siguiente registro contable en
el Libro de Diario:
Fecha
01/11/20xx

Cdigo

Nombre de la Cuenta
Caja
Maquinaria y Equipo
Capital suscrito y pagado

Debe
600.000.000
200.000.000

Haber

800.000.000

b) La consignacin bancaria, da lugar al siguiente asiento:


Fecha
01/11/20xx

Cdigo

Nombre de la Cuenta
Bancos
Caja

Debe
600.000.000

Haber
600.000.000

c) Tenemos aqu, una compra de muebles y enseres a crdito:


Fecha
02/11/20xx

Cdigo

Nombre de la Cuenta
Equipo de Oficina
Cuentas corrientes comerciales

Debe
15.000.000

Haber
15.000.000

d) Una transaccin por Gasto de mantenimiento


Fecha

Cdigo

Nombre de la Cuenta

Debe

Haber

Para ver trabajos similares o recibir informacin semanal sobre nuevas publicaciones, visite www.monografias.com

www.monografias.com

03/11/20xx

Mantenimiento y Reparaciones
Bancos

1.000.000
1.000.000

e) Una transaccin por Compra de mercancas a crdito


Fecha
09/11/20xx

Cdigo

f)
Fecha
10/11/20xx

Nombre de la Cuenta
De Mercancas
Proveedores Nacionales

Debe
110.000.000

Haber
110.000.000

Una transaccin por Venta de mercancas a crdito


Cdigo

Nombre de la Cuenta
Clientes
Comercio al P/M y p/m

Debe
15.000.000

Haber
15.000.000

g) Una transaccin por Salarios de Personal operativo


Fecha
19/11/20xx

Cdigo

Nombre de la Cuenta
Gastos de Personal
Bancos

Debe
6.000.000

Haber
6.000.000

h) Una transaccin por Cargo a deudas


Fecha
26/11/20xx

Cdigo

i)
Fecha
29/11/20xx

Fecha
30/11/20xx

Debe
90.000.000

Haber
90.000.000

Una transaccin por Prstamo recibido


Cdigo

j)

Nombre de la Cuenta
Proveedores Nacionales
Bancos

Nombre de la Cuenta
Bancos
Nacionales

Debe
40.000.000

Haber
40.000.000

Una transaccin por Gasto de arrendamiento


Cdigo

Nombre de la Cuenta
Arrendamiento
Bancos

Debe
2.000.000

Haber
2.000.000

k) Una transaccin por Sueldos de personal administrativo


Fecha
30/11/20xx

Cdigo

l)
Fecha
30/11/20xx

Nombre de la Cuenta
Gastos de Personal
Bancos

Debe
8.000.000

Haber
8.000.000

Una transaccin por Compra de mercancas a crdito


Cdigo

Nombre de la Cuenta
Cuentas Corrientes Comerciales
Bancos

Debe
15.000.000

Haber
15.000.000

3- Resuelva los siguientes ejercicios hasta obtener el Balance de Comprobacin:


a) La empresa Miscelnea, dedicada al negocio de ropa inicia operaciones el 1 de Enero, con
los siguientes valores: Caja $ 3.000.000, Bancos $ 450.000, Mercancas $ 2,500.000,

Para ver trabajos similares o recibir informacin semanal sobre nuevas publicaciones, visite www.monografias.com

www.monografias.com

Clientes $ 600.000, Equipos de oficina $ 500.000, Proveedores $ 680.000, Acreedores


Varios $ 1.500.000
b) En enero 4 compra mercancas por $ 900.000, se cancela $ 225.000 con cheques y el resto
se queda debiendo
c) En enero 5 un cliente cancela la suma de $ 300.000
d) En enero 5, se consigna en el Banco $ 750.000
e) En enero 8, se abona a los proveedores la suma de $ 380.000
f) En enero 10, se compra papelera y tiles por $ 20.000 que se cancelan en efectivo
g) En enero 15 se recoge una letra a cambio de un cheque por valor de $ 500.000
Solucin: En este ejercicio se comienza con el Balance de Apertura o Balance Anterior, luego
siguen los Asientos de Diario y su Resumen o Comprobante de Diario, antes de completar la
confeccin del Balance de Comprobacin de saldos, as:
a) Balance Anterior

b) Asientos de Diario
Fecha
04/01/20xx

Fecha
05/01/20xx

Fecha
05/01/20xx

Fecha

Cdigo
6205
1110
2205

Nombre de la Cuenta
De Mercancas
Bancos
Proveedores Nacionales

Debe

Cdigo
1105
1305

Nombre de la Cuenta
Caja
Clientes

Debe

Cdigo
1110
1105

Nombre de la Cuenta
Bancos
Caja

Debe

Cdigo

Nombre de la Cuenta

Debe

Haber
900.000
225.000
675.000

Haber
300.000
300.000

Haber
750.000
750.000

Haber

Para ver trabajos similares o recibir informacin semanal sobre nuevas publicaciones, visite www.monografias.com

www.monografias.com

08/01/20xx

Fecha
10/01/20xx

Fecha
15/01/20xx

2205
1110

Proveedores Nacionales
Bancos

380.000

Cdigo
5195
1105

Nombre de la Cuenta
Diversos
Caja

Debe

Cdigo
2380
2305

Nombre de la Cuenta
Acreedores Varios
Cuentas Corrientes Comerciales

Debe

380.000

Haber
20.000
20.000

Haber
500.000
500.000

c) Comprobantes de Diario

d) Balance de Comprobacin

Para ver trabajos similares o recibir informacin semanal sobre nuevas publicaciones, visite www.monografias.com

www.monografias.com

4- La concesionaria Samsung Ltda. fue organizada el 1 de Abril y su negocio consiste en la


reparacin de electrodomsticos de marca. Confeccione los asientos de diario de las siguientes
transacciones de la empresa durante la primera semana del mes y prepare un Resumen o
Comprobante de Diario:
a) La sociedad se constituyo con un capital de $ 1.000.000.000 mediante aportes por partes
iguales entre los socios Rafael Valadon, Pedro Infante, Helena Vargas, Miguel Meja y Gina
Lollobrigida
b) La empresa consign la mitad en el Banco de Crdito y la otra mitad en el Banco de
Bogot.
c) Alquil una oficina en el Parque Central y otra en el Country Plaza, pag respectivamente $
400.000 con cheque del Banco de Crdito y $ 600.000 con cheque del Banco de Bogot,
por la primera semana de arrendamiento,
d) Compr a Ferretera Kike 5 kits de reparaciones a domicilio nuevos a $ 210.000 cada uno,
pagando el 30% al contado y el resto a 60 das y tambin, a Panamericana S.A. equipo de
oficina, entregando $ 1.000.000 en efectivo y una letra por $ 3.000.000, ambos con cheque
del Banco de Bogot.
e) Pag sueldos de $ 600.000 al Jefe de Taller y $ 700.000 al Jefe de Ventas, contrato
publicidad con el Heraldo y Telecaribe por tres millones cada uno a 90 das y pag $
1.000.000 al primero y $ 2.000.000 al segundo con cheques del Banco de Crdito
f) Abri una cuenta en Supertaxis Ejecutivo Calle 53 y otra en Taxi Paraso. Estas prestaran
servicios a los cinco tcnicos de mantenimiento electrnico, todo por un costo de $ 400.000
mensuales anticipados cada una
g) Hizo dos reparaciones durante el da cuarto y recibi por ello $ 300.000 en efectivo. A
excepcin del transporte, la empresa cobra $ 75.000 fijos por el diagnostico, limpieza y
ajustes y los dems gastos del servicio (materiales, repuestos, etc.) los paga el cliente.
h) Prest servicios de mantenimiento a la CUC y la Unisimonbolivar por valor de $ 2.300.000
a cada una, donde fueron enviados 4 tcnicos por dos das.. Estos convenios establecen el
pago cada final de mes.
i) La compra de repuestos a la Samsung Bogot, tuvo un costo de $ 960.000, pagados con
cheque del Banco de Bogot.
j) Prest tres servicios domiciliarios el sexto da y recibi por ello $ 405.000, incluyendo la
colocacin de repuestos por $ $ 180.000
k) Pag la segunda semana al Jefe de Taller y al Jefe de Ventas con cheques del Banco de
Crdito
l) Pag $ 125.000 de cuota de seguridad y aseo por cada uno de los locales con cheque del
Banco de Crdito.
Solucin: En este ejercicio se ilustra la utilizacin de subcuentas y auxiliares
a) Esta es una transaccin de Aportes de Capital, as:
Fecha
01/04/20xx

Cdigo
1105
3115
310505
31050501
31050502
31050503
31050504
31050505

Nombre de la Cuenta
Caja
Aportes sociales
Cuotas o partes de inters social
Rafael Valadon
Pedro Infante
Helena Vargas
Miguel Meja
Gina Lollobrigida

Debe
1.000.000.000

Haber

200.000.000
200.000.000
200.000.000
200.000.000
200.000.000

Observe que la anotacin de las cifras tiene lugar al nivel catalogado con el mayor numero de
dgitos.
b) Esta es una transaccin de consignacin en entidades financieras, as:
Fecha
01/04/20xx

Cdigo
1110
111005
11100501

Nombre de la Cuenta
Bancos
Moneda nacional
Banco de Crdito

Debe

Haber

500.000.000

Para ver trabajos similares o recibir informacin semanal sobre nuevas publicaciones, visite www.monografias.com

www.monografias.com

11100502
1105

Banco de Bogot
Caja

500.000.000
1.000.000.000

c) Esta es una simple transaccin de Gastos, as:


Fecha
02/04/20xx

Cdigo
5120
1110
111005
11100501
11100502

Nombre de la Cuenta
Arrendamientos
Bancos
Moneda nacional
Banco de Crdito
Banco de Bogot

Debe
1.000.000

Haber

400.000.000
600.000.000

d) Esta es una transaccin de Compra a crdito de activos diferentes a mercancas:


Fecha
02/04/20xx

Cdigo
1520
1524
1110
111005
11100502
2305
230501
230502

Nombre de la Cuenta
Maquinaria y Equipo
Equipo de Oficina
Bancos
Moneda nacional
Banco de Bogot
Cuentas Corrientes Comerciales
Ferretera Kike
Panamericana S.A.

Debe
1.050.000
4.000.000

Haber

1.315.000
735.000
3.000.000

e) Esta es una transaccin de Compra a crdito de activos diferentes a mercancas:


Fecha
03/04/20xx

Cdigo
5105
5205
5235
1110
111005
11100501
2305
230511
230512
f)

Fecha
03/04/20xx

Nombre de la Cuenta
Gastos de Personal
Gastos de Personal
Servicios
Bancos
Moneda nacional
Banco de Crdito
Cuentas Corrientes Comerciales
El Heraldo
Telecaribe

Debe

Haber

600.000
700.000
6.000.000

4.300.000
2.000.000
1.000.000

Esta es una transaccin de Gastos pagados por anticipado:


Cdigo
5105
510501
510502
1110
111005
11100501

Nombre de la Cuenta
Gastos pagados por anticipado
Supertaxi Ejecutivo
Taxi Paraiso
Bancos
Moneda nacional
Banco de Crdito

Debe

Haber
400.000
400.000

800.000

g) Esta es una transaccin de Venta y reparacin de efectos personales y electrodomesticos


al contado:
Fecha
04/04/20xx

Cdigo
4135
4235
1105

Nombre de la Cuenta
Comercio al P/M y al p/m
Servicios
Caja

Debe

Haber
150.000
150.000
300.000

h) Esta es una transaccin de Venta de servicios a crdito:

Para ver trabajos similares o recibir informacin semanal sobre nuevas publicaciones, visite www.monografias.com

www.monografias.com

Fecha
04/04/20xx

Cdigo
4235
1305
130501
130502
i)

Fecha
04/04/20xx

Fecha
07/04/20xx

Debe
4.600.000

Haber

2.300.000
2.300.000

Esta es una transaccin de Compra de mercancas al contado:


Cdigo
6105
1110
111005
11100501

j)

Nombre de la Cuenta
Servicios
Clientes
CUC
Unisimonbolivar

Nombre de la Cuenta
De Mercancas
Bancos
Moneda nacional
Banco de Bogot

Debe

Haber
960.000

960.000

Esta es una transaccin de Venta de Mercancas y reparaciones al contado:


Cdigo
4105
4235
1105

Nombre de la Cuenta
De Mercancas
Servicios
Caja

Debe

Haber
180.000
225.000
405.000

k) Esta es una transaccin de Gastos al contado:


Fecha
07/04/20xx

Cdigo
5105
5205
1110
111005
11100501
l)

Fecha
03/04/20xx

Nombre de la Cuenta
Gastos de Personal
Gastos de Personal
Bancos
Moneda nacional
Banco de Crdito

Debe

Haber
600.000
700.000

1.300.000

Esta es una transaccin de Gastos al contado:


Cdigo
5135
1110
111005
11100501

Nombre de la Cuenta
Servicios
Bancos
Moneda nacional
Banco de Crdito

Debe

Haber
125.000

125.000

5- Son tan numerosas las transacciones del negocio del Seor Johnny Piescalientes que para su
mejor control requiere la elaboracin diaria de resmenes y balances de comprobacin.
Partiendo del Balance de Apertura estudiado en la unidad anterior, elaborar: los asientos de
diario, el Comprobante de Diario y el Balance de Comprobacin, sabiendo que durante el primer
da de labores en lunes 4 de enero el negocio del seor Johnny Piescalientes ha realizado las
siguientes transacciones:
Asiento
de
Diario N
1

OPERACIONES EFECTUADAS
por el Seor Johnny Piescalientes el Lunes 4 de Enero,
para ser contabilizadas por los estudiantes
Lunes 4 de Enero
Asiento de Apertura de los Libros del empresario Johnny
Piescalientes en el libro diario, soportado con el Inventario
General de Bienes y Obligaciones y, en el Libro mayor
con soporte del Comprobante Diario N 001
Lunes 4 de Enero

Valores

131.353.661

Para ver trabajos similares o recibir informacin semanal sobre nuevas publicaciones, visite www.monografias.com

www.monografias.com

2
3
4

8
9
10

Pagamos abono anticipado a la Factura 335 de


Federaltex
Pagado el arrendamiento del local con cheque N 001 del
Banco de Bogot
Venta al contado de lo siguiente:
400 m. tela estampada Coltejer a $ 7000/m. 2.800.000
300 m. de dril naval blanco a $ 33645/m.
1.009.350
80 pares medias GEF para nios a $ 5400
432.000
20 cajas de pauelos Pyramide a $72080
1.441.600
Consignado hoy en los Bancos:
Banco de Bogot
2.400.000
Banco Mercantil
1.600.000
Compra a Pedro Castro, Bucaramanga, Fact. N 176
300 pares de zapatos Corona a $ 39000/par, pagaderos,
la mitad con cheque N 002 del Banco de Bogot a la
fecha y el resto a 60 das.
Compra a lvaro Vargas de un Software Contable
Comodn N 47345 para uso de las oficinas y pagada con
cheque del Banco Mercantil N 001 por suma de
Cancel Pedro Yudex Factura 88 con descuento 2%
Consignacin en la fecha en Banco de Bogot y Banco
Mercantil por mitades la suma de
Pagado en efectivo a Telecaribe por el aviso publicado

448.269
3.000.000
5.682.950

4.000.000

11.700.000

218.000

5.820.000
5.500.000
954.681

Solucin: En este ejercicio se ilustra la opcin de los comerciantes de hacer imprimir libros
diarios con tres columnas, en cuyo caso, las anotaciones sobre las subcuentas y auxiliares, son
efectuadas en la columna de Parciales.

Para ver trabajos similares o recibir informacin semanal sobre nuevas publicaciones, visite www.monografias.com

www.monografias.com

Para ver trabajos similares o recibir informacin semanal sobre nuevas publicaciones, visite www.monografias.com

www.monografias.com

Para ver trabajos similares o recibir informacin semanal sobre nuevas publicaciones, visite www.monografias.com

www.monografias.com

La evaluacin se distribuye entre evaluaciones individuales en clase y la presentacin de un panel


acerca de una situacin problmica.
2.6.2. Problemas Propuestos
1- La partida doble tiene fundamento cuando una persona:
a) Vende y otra compra
b) Recibe y anota las mercancas
c) Entrega un recibo, otorga una factura
d) Sale favorecido, reclama un premio
e) Encuentra un recibo, va a pagarlo
2- En un asiento contable compuesto participan:
a) Dos cuentas que se acreditan
b) Dos cuentas que se debitan
c) Tres cuentas que se debitan
d) Ms de dos cuentas
e) Una se debita y dos se acreditan
3- Cuentas reales son las que aparecen en el:
a) Estado de Flujo de Efectivo
b) Estado de Situacin Financiera
c) Estado de Resultado
d) Balance General
4- Cada una de las partidas siguientes se encuentra en los Estados Financieros. Indique en cual
(B: Balance General, R: Estado de Resultados, o F: Flujo de Efectivo)
Gasto de publicidad
Inventario de ACPM
Iva por pagar
Venta de gasolina
Gastos de fundacin

Inversiones
Ttulos valores
Cuentas por pagar
Gasto operacional
Ctas. Ctes. Ciales.

Propiedad Planta y E
Gastos pag anticipad
Obligaciones Financ.
Ingresos Financieros
Prstamo a emplead

5- El Capital de personas naturales es una cuenta de:


a) Activo
b) Pasivo
c) De orden

Para ver trabajos similares o recibir informacin semanal sobre nuevas publicaciones, visite www.monografias.com

www.monografias.com

d) Reserva
e) Patrimonio
6- Los Activos se encuentran conformados por las siguientes cuentas:
a) Disponible, capital, reservas
b) Disponible, deudores, utilidad
c) Disponible, inventarios y capital
d) Disponible, deudores, inversiones e inventarios
e) Disponible, clientes y proveedores
7- Cada una de las partidas siguientes se encuentra en los Estados Financieros. Indique en cual
(B: Balance General, R: Estado de Resultados, o F: Flujo de Efectivo)
Gasto de publicidad
Inventario de ACPM
Iva por pagar
Venta de gasolina
Gastos de fundacin
Capital de pers. nat.
Efectivo
Inventario de materiales

Inversiones
Ttulos valores
Cuentas por pagar
Gasto operacional
Ctas. Ctes. Ciales.
Honorarios ganados
Terrenos
Pago por servicios
pblicos

Propiedad Planta y E
Gastos pag anticipad
Obligaciones Financ.
Ingresos Financieros
Prstamo a emplead
Salarios por pagar
Pago de sueldos
Compra de material de
consumo

8- Si hubo ingresos por $ 50.000.000, gastos por $ 35.000.000 y retiros del propietario por $
10.000.000, la utilidad o prdida operacional del negocio ser:
a) $ 45.000.000
b)
15.000.000
c)
35.000.000
d)
5.000.000
9- Jorge Noriega administra su propio negocio de restaurantes. A continuacin se muestra un
resumen del mes de Agosto en forma de ecuacin. Cada rengln indica el efecto de una
transaccin sobre la ecuacin. Describa a que pudo haberse referido cada transaccin.

10- Indique el efecto de cada uno de los siguientes hechos contables sobre la ecuacin contable,
escribiendo a la derecha de cada hecho la letra correspondiente tomada de la siguiente lista:
A) Aumento de un activo y disminucin de otro
B) Aumento de un activo y de un pasivo
C) Aumento de un activo y del capital contable
D) Disminucin de un activo y de un pasivo
E) Disminucin de un activo y del capital contable

Para ver trabajos similares o recibir informacin semanal sobre nuevas publicaciones, visite www.monografias.com

www.monografias.com

Los hechos contables fueron:


a)
b)
c)
d)
e)
f)
g)
h)
i)
j)

Cargo de un cliente que paga en efectivo la suma de $ 630.200


Abono a acreedores por $ 380.000
Inversin adicional del propietario por $ 250.000 en efectivo
Pago de publicidad por $ 437.500 en efectivo
Cargo a clientes del servicio de encomienda por $ 29.000
Materiales de consumo comprados al contado por $ 750.000
Causacin del alquiler actual y pago del siguiente mes por $ 250.000
Efectivo recibido de clientes por $ 900.000
Efectivo entregado al propietario para su uso personal
El inventario de materiales de consumo en existencia al final de mes fue de $
300.000, por consiguiente se haban utilizado $ 450.000

11- En 2 de Marzo se abri un negocio comercial denominado La Especial. Confeccione los


asientos de diario de las siguientes transacciones y prepare un Resumen o Comprobante de
Diario, con base en los siguientes valores:
a) El Balance de Apertura muestra la cuenta de Caja $ 180.000, Bancos $ 600.000,
Mercancas $ 6.000.000, Clientes $ 1.200.000, Equipo de oficina $ 1.330.000, Proveedores
$ 1.200.000 y Cuentas por Pagar $ 1.330.000
b) En marzo 5 compra a crdito nuevo Equipo de Oficina por $ $ 1.500.000
c) En marzo 8, un cliente cancela la suma de $ 500.000
d) En marzo 11, se consigna en el banco la suma de $ $ 522.000
e) En marzo 15 se recoge una letra de cambio por $ 630.000
f) En marzo 18 se compra un vehculo para uso de la empresa por $ 90.000.000, habiendo
pagado15.millones en cheque y cinco letras bimestrales cada una por igual valor
g) En marzo 21 se compran al contado, tiles de oficina por $ 60.000.000
h) En marzo 31, el propietario del negocio aporta otros $ 15.000.000 en efectivo
i) Ese mismo da, se consignan en el banco la suma de $ 15.000.000
12- Partiendo del Balance de Apertura del Centro Comercial Snchez Escolar estudiado en la
unidad anterior y sabiendo que ha realizado durante el mes de Septiembre las siguientes
transacciones, elaborar: los asientos de diario, el Comprobante de Diario y el Balance de
Comprobacin.
a) Septiembre 1: los socios efectan nuevos aportes mediante Escritura Pblica No. 4868
de la Notara Quinta de Barranquilla con los siguientes datos:
El socio JAVIER SANCHEZ aporta $ 2.500.000 con cheque No. 8543381 del
Bancolombia segn Recibo de Caja No. 0001 y muebles para oficina por valor de $
1.800.000, segn Factura No. LB 450
La socia IVANOVA ESCOLAR aporta
mercanca por valor de $ 1.200.000 segn Factura No. 320 y $ 2.000.000 con cheque
No. 239045 del BBVA segn Recibo de Caja No. 0002.
b) Septiembre 4: La empresa adquiere a crdito una motocicleta para el reparto de
mercancas, con la empresa china Jing Cheng, con domicilio en Barranquilla, por valor
de $ 5.200.000, mediante la Factura CH 569, a 20 meses de plazo sin recargo de
financiaciones.
c) Septiembre 4: Tiene lugar la primera consignacin oficial en la cuenta 1576 del Banco
Popular, a nombre de la empresa, por valor de $ 1.287.530.
d) Septiembre 10. Compra a crdito de Harina Yacuanquer, a la empresa Distrinar S.A. ,
20 sacos a $ 77310 cada saco, segn Factura No. 7980
e) Septiembre 10. La empresa obtiene un prstamo bancario para adelantar trmites
oficiales, por valor de $ 500.000 al 3% mensual, intereses anticipados.
f) Septiembre 10. Se recibe la Nota Dbito del Banco Popular por valor de los intereses
anticipados del primer mes del Prstamo.
g) Septiembre 15. Se realiza la primera venta que hace el negocio, en efectivo, de 4 sacos
de caf, con un margen de ventas del 23 %

Para ver trabajos similares o recibir informacin semanal sobre nuevas publicaciones, visite www.monografias.com

www.monografias.com

h) Septiembre 15. Se recibe un pedido de 4 sacos de caf, pagado por anticipado por
valor de $ 2.150.000 de parte del Hotel Ambassador.
i) Septiembre 15. Aceptacin del compromiso con Aseo Tcnico S.A. para que se
encargue de la vigilancia y el aseo de la empresa, por $ 300.000. El pago se har los
das 30 de cada mes.
j) Septiembre 22. Tiene lugar la primera venta a Crdito de 25 sacos de arroz a
Hipervianda, mediante Factura No. 3467, a 30 das de plazo.
k) Septiembre 22. Se paga la primera cuota de la moto, por $ 260.000 a Jing Cheng S.A.,
segn cheque No 007 del Banco Popular.
l) Septiembre 23. Hipervianda abona la suma de $ 1.000000 a su Factura No 3467.
m) Septiembre 24. Compramos al contado 400 litros de aceite de oliva, nueva lnea de
productos, a Los Olivares SCA y se paga con cheque No. 009 del Banco Popular
n) Septiembre 24. Las ventas por catlogo convenidas con Yanbal S.A, proporciona los
primeros $ 600.000 de ingresos por comisiones del 10% en ventas
o) Septiembre 28. Se adquiere el programa Fomplus de Contabilidad sistematizada a la
empresa Computer Ltda, por valor de $ 3.000.000 amortizables a 20 meses, segn su
Factura No. 01 y pagamos con cheque No. 011 del Banco Popular.
p) Septiembre 28. El Trabajador Elkin Rada, requiere apoyo de la empresa por valor de $
4.500.000, para atender calamidades ocasionadas por un tornado en su lugar de
residencia en la poblacin de Baranoa.
q) Septiembre 30. La empresa decide pagar la totalidad del prstamo recibido del Banco
popular.
13- Son tan numerosas las transacciones del negocio del Seor Johnny Piescalientes que para su
mejor control requiere la elaboracin diaria de resmenes y balances de comprobacin.
Partiendo del Balance de Apertura estudiado en la unidad anterior, elaborar: los asientos de
diario, el Comprobante de Diario y el Balance de Comprobacin, sabiendo que durante el primer
da de labores en martes 5 de enero el negocio del seor Johnny Piescalientes ha realizado las
siguientes transacciones:

Para ver trabajos similares o recibir informacin semanal sobre nuevas publicaciones, visite www.monografias.com

www.monografias.com

14- Son tan numerosas las transacciones del negocio del Seor Johnny Piescalientes que para su
mejor control requiere la elaboracin diaria de resmenes y balances de comprobacin.
Partiendo del Balance de Apertura estudiado en la unidad anterior, elaborar: los asientos de
diario, el Comprobante de Diario y el Balance de Comprobacin, sabiendo que durante el primer
da de labores en mircoles 6 de enero el negocio del seor Johnny Piescalientes ha realizado
las siguientes transacciones:

Para ver trabajos similares o recibir informacin semanal sobre nuevas publicaciones, visite www.monografias.com

www.monografias.com

15- Son tan numerosas las transacciones del negocio del Seor Johnny Piescalientes que para su
mejor control requiere la elaboracin diaria de resmenes y balances de comprobacin.
Partiendo del Balance de Apertura estudiado en la unidad anterior, elaborar: los asientos de
diario, el Comprobante de Diario y el Balance de Comprobacin, sabiendo que durante el primer
da de labores en jueves 7 de enero el negocio del seor Johnny Piescalientes ha realizado las
siguientes transacciones:

Para ver trabajos similares o recibir informacin semanal sobre nuevas publicaciones, visite www.monografias.com

www.monografias.com

16- Son tan numerosas las transacciones del negocio del Seor Johnny Piescalientes que para su
mejor control requiere la elaboracin diaria de resmenes y balances de comprobacin.
Partiendo del Balance de Apertura estudiado en la unidad anterior, elaborar: los asientos de
diario, el Comprobante de Diario y el Balance de Comprobacin, sabiendo que durante el primer
da de labores en viernes 8 de enero el negocio del seor Johnny Piescalientes ha realizado las
siguientes transacciones:

Para ver trabajos similares o recibir informacin semanal sobre nuevas publicaciones, visite www.monografias.com

www.monografias.com

17- La Lavandera La Automtica presenta el siguiente Balance de Apertura:

Para ver trabajos similares o recibir informacin semanal sobre nuevas publicaciones, visite www.monografias.com

www.monografias.com

Adems, durante el resto del mes de Enero efectu las siguientes transacciones:
a) Enero 9: Pag $ 1.500.000 por concepto de arrendamiento por el mesen curso,
b) Enero 10: Recibi $ 1.000.000 por concepto de servicios de lavandera vendidos,
c) Enero 12: Pag $ 800.000 por concepto de gastos de propaganda,
d) Enero 12: Pag $ 75.000 por concepto de Materiales para el consumo de la lavandera,
e) Enero 14: Envi una Factura a la Ca Solar por $ 1.200.000 por servicios prestados,
f) Enero 15: Pag $ 400.000 por concepto de salarios,
g) Enero 20: Recibi $ 300.000 por concepto de servicios a la Ca Lunar,
h) Enero 22: Recibi el dinero que la Ca Solar le adeudaba,
i)
Enero 24: Recibi $ 1.100.000 en efectivo de la Ca Venusiana por concepto de servicios,
j)
Enero 28: Envi factura a Marte Ltda. por concepto de servicios por $ 1.300.000,
k) Enero 31: Pag $ 700.000 por concepto de salarios,
l)
Enero 31: Pag $250.000 por concepto de servicio telefnico.
Se pide hacer los Asientos de Diario, Comprobante de Diario, Libro Mayor (cuentas T), Balance
de Comprobacin.
18- El Balance General de la Ca Picapiedra S.A. despus del cierre en 30 de Abril fue:

Para ver trabajos similares o recibir informacin semanal sobre nuevas publicaciones, visite www.monografias.com

www.monografias.com

Durante el mes de Mayo, la Compaa hizo las siguientes transacciones:


a) Mayo 1: Pag un mes de alquiler del almacn por $ 3.000.000
b) Mayo 5: Vendi a XYZ Mercancas a Crdito por $ 8.000.000
c) Mayo 10: Compr Mercancas en efectivo por $ 5.500.000
d) Mayo 15: Vendi Mercancas en Efectivo por $ 3.000.000
e) Mayo 2 0: Pag los $ 2.000.000 de impuestos a la Renta
f) Mayo 25: Pag varias cuentas del mes anterior por $ 12.000.000
g) Mayo 31: Compr a DEF Mercancas a Crdito por $ 9.000.000
Se pide hacer los Asientos de Diario, Comprobante de Diario, Balance de Comprobacin.
19- Partiendo del Balance General de la Ca Picapiedra S.A. obtenido en el problema anterior,
confeccionarlo al siguiente mes despus de las siguientes transacciones::
a) Junio 2: Cobr varias cuentas por $ 15.000.000
b) Junio 7: Pag a DEF la ltima factura anterior
c) Junio 12: Compr en efectivo tiles de Oficina por $ 1.500.000
d) Junio 17: Hizo ventas en efectivo por $ 14.000.000
e) Junio 22: Compr Mercancas en Efectivo por $ 8.500.000
f) Junio 26: Pag Telfono por $ 350.000
g) Junio 28: Pag Publicidad a El Heraldo por $ 1.400.000
h) Junio 30: Pag Salarios por $ 5.000.000
Se pide hacer los Asientos de Diario, Comprobante de Diario, Balance de Comprobacin.
20- En 30 de Abril, despus de cerrar los libros, el libro Mayor de la Ca. CURRAMBA DE
ELECTRODOMSTICOS S.A. tena las siguientes cuentas:

Para ver trabajos similares o recibir informacin semanal sobre nuevas publicaciones, visite www.monografias.com

www.monografias.com

Las transacciones de esta compaa durante el mes de Mayo se pueden resumir as:

Se requiere:
Hacer los asientos para registrar las transacciones que se describen en los numerales 1 al 6,
haciendo un asiento compuesto para cada numeral.
a) Se pide hacer los Asientos de Diario, Comprobante de Diario, Balance de Comprobacin.
El ciclo contable
3.1. El Periodo Contable

Para ver trabajos similares o recibir informacin semanal sobre nuevas publicaciones, visite www.monografias.com

www.monografias.com

El periodo contable es uno de los principios de Contabilidad Generalmente Aceptados y se refiere a que las
operaciones econmicas de una empresa se deben registrar y reconocer en un determinado tiempo. Los
ingresos, gastos, costos e impuestos deben registrarse en el perodo econmico en que ocurren y
reconocerse en el periodo contable en que se causan.
La existencia del periodo contable, hace que se cumpla uno de los principales objetivos de la contabilidad
que es su utilidad, al permitir la comparacin de los desempeos que ha tenido la empresa en diversos
periodos. Es esta comparacin la que permite analizar la informacin contable, para con base a ella tomar
las decisiones econmicas y financieras. Los anlisis contables comparativos, adems de ayudar al
conocimiento de la realidad de la empresa como diagnostico, permite proyectarla con base a los
antecedentes que seala el periodo contable.
El periodo contable tambin guarda relacin con el periodo fiscal de la regulacin tributaria. En el campo
fiscal, existen diferentes periodos dependiendo el impuesto. En Colombia, el impuesto de renta tiene un
periodo anual, el impuesto sobre las ventas (Iva) es bimestral, el Impuesto de industria y comercio en la
mayor parte de los municipios es bimestral, el Gravamen a los movimientos financieros se presenta
semanal, la retencin en la fuente es de periodo mensual.
Claro est que en el caso de que existan errores, estos se pueden corregir mediante los respectivos ajustes
aplicados al periodo o ao en que se adviertan. Los ingresos, gastos e impuestos de perodos anteriores no
registrados en su oportunidad deben contabilizarse en cuentas independientes a las de las correspondientes
al perodo econmico en que se detecten, con el fin de dejar en claro que es lo correspondiente de cada
periodo y para no desfigurar los resultados que pueden afectar los anlisis y dems estudios que se
requieran hacer.
Cuando nos referimos que el periodo contable es por lo general de un ao, significa que no siempre debe
ser ese lapso de tiempo, puesto que se puede dar el caso de las empresas que inician operaciones dentro
del ao, caso el cual, el periodo ser desde la fecha en que se iniciaron operaciones hasta el 31 de
diciembre el respectivo ao. Igual sucede con las empresas que se liquidan dentro del mismo ao, en cuyo
caso el periodo, tanto contable como fiscal, corresponde desde el 1 de enero hasta la fecha en que se
realiza la liquidacin del ente.
3.2. Asientos de Ajustes
Se entiende por Ajuste, el asiento contable necesario para llevar el saldo de una cuenta a su valor real. Al
finalizar el periodo contable, las cuentas deben presentar su saldo real, por cuanto estos valores servirn de
base para preparar estados financieros. Cuando los saldos de las cuentas no son reales es necesario
aumentarlos, disminuirlos o corregirlos mediante un asiento contable llamado asiento de ajuste.
Debemos utilizar asientos de ajuste siempre que cualquier transaccin afecte los ingresos o los gastos
durante ms de un periodo contable, estos asientos se podran hacer de forma diaria, pero por asuntos
prcticos estos deben hacerse mejor cada final de periodo.
El nmero de ajustes que se pueden hacer al finalizar el mes son innumerables, pero los ms importantes
caen dentro de cuatro categoras:
AJUSTES PARA DESGLOSAR EL INGRESO CAUSADO
Este es el clsico ejemplo de ventas hechas por anticipado, en el que los ingresos recibidos por adelantado
representan un pasivo diferido. Por ejemplo, en el caso de las contratos de obra, cuando se le ha cancelado
el total de la obra al contratista sin haberla terminado, en cada periodo posterior, este tiene que considerar
como ingreso del periodo nicamente la parte proporcional del avance efectuado y, el saldo de dichos
ingresos, continuara registrado en la cuenta de pasivos diferidos hasta la total culminacin de la obra.
Ejemplo: Si el 28 de Agosto de 20xx la empresa Vigilancia del Norte SCS., recibe del Almacn Tierra Santa
Ltda., la suma de $ 360.000 por concepto de los servicios de seguridad nocturna de los meses de
septiembre, octubre y noviembre, este da, la empresa receptora de los dineros, registra el siguiente asiento:
Fecha
28/08/20xx

Cdigo
1105
2705

Nombre de la Cuenta
Caja
Ingresos Rec. por Anticipado

Debe
360.000

Haber
360.000

Ejemplo: Hacer el asiento de ajuste al vencimiento del primer mes de contratacin.


Si el 28 de agosto de 20xx el valor de $ 360.000 recibidos por la empresa Vigilancia del Norte SCS.,
corresponden a tres meses de seguridad nocturna sobre el local del almacn Tierra Santa Ltda., el valor
causado por la prestacin de los servicios durante el primer mes es:

Para ver trabajos similares o recibir informacin semanal sobre nuevas publicaciones, visite www.monografias.com

www.monografias.com

Fecha
30/09/20xx

Cdigo
2705
4155

Nombre de la Cuenta
Ingresos Rec. por Anticipado
Actividades Inmobiliarias

Debe
120.000

Haber
120.000

AJUSTE PARA REGISTRAR EL NO PAGO DE UNA OBLIGACION


Ahora, en contrario, as como una empresa puede recibir dineros anticipado, tambin puede pagar
anticipadamente por servicios, arrendamientos, comisiones y otros. Cuando la empresa ha devengado un
ingreso y no se ha cobrado, el valor de ingreso ya causado se convierte en un derecho de la empresa; por
ello debe realizar un ajuste por el valor correspondiente, debitando una cuenta de Activos denominada
Ingresos por Cobrar y acredita la cuenta del ingreso respectivo.
Otro caso muy comn para este tipo de ajuste es el de las ventas que ya se han efectuado a los clientes,
pero que aun no han sido cobradas, generalmente se trata de ventas que se han hecho sobre una cuota
inicial como es el caso de los artefactos elctricos u otros tipos de insumos.
Ejemplo: La empresa Pedro Picala e Hijos Ltda., presta el servicio de arrendamiento de fotocopiadoras a
otras empresas. Si al finalizar el periodo contable, la empresa Pinturas Arte y Decoracin Ltda. no ha
cancelado el valor de un mes del servicio por $ 500.000, el arrendador efecta el siguiente asiento:
Fecha
31/08/20xx

Cdigo
1345
4220

Nombre de la Cuenta
Ingresos por Cobrar
Arrendamientos

Debe
500.000

Haber
500.000

AJUSTES PARA REGISTRAR LOS GASTOS ACUMULADOS


El ejemplo ms comn de este tipo de asiento de ajuste son aquellos por los cuales una empresa se ha
incurrido en un gasto y no se ha contabilizado. El valor del gasto ya causado se convierte en un pasivo para
la empresa y debe realizarse un ajuste por el valor correspondiente. Para ello, debita la cuenta del gasto
correspondiente y acredita la cuenta de Costos y gastos por pagar.
Ejemplo: Al finalizar el mes de septiembre, a la Urbanizadora Vista al Mar Ltda., no le han llegado las
cuentas de cobro por los servicios pblicos por cuya razn hace un estimado de las cuentas pendientes de
pago as: AAA S.A. por servicios de Agua $ 488.300, Telmex por servicios de telfono $ 125.000 y
Electricaribe por servicios de energa $287.200 y registra:
Fecha
31/09/20xx

Cdigo
5135
2335
233521
233522
233523

Nombre de la Cuenta
Servicios
Costos y Gastos por pagar
AAA S.A,
Telmex S.A.
Electricaribe S.A.

Debe
900.500

Haber

488.300
125.000
287.200

Ejemplo: Si el dia 4 de Octubre se recibe la cuenta de cobro de AAA S.A. por valor de $ 480.000, la empresa
Urbanizadora Vista al Mar debe hacer el siguiente ajuste:

Fecha
04/10/20xx

Cdigo
2335
233521
5135
1110

Nombre de la Cuenta
Costos y Gastos por pagar
AAA S.A,
Servicios
Bancos

Debe

Haber

488.300
8.300
480.000

Para ver trabajos similares o recibir informacin semanal sobre nuevas publicaciones, visite www.monografias.com

www.monografias.com

Ejemplo: Si el dia 7 de Octubre se recibe la cuenta de cobro de Electricaribe S.A. por valor de $ 300.000, la
empresa Urbanizadora Vista al Mar debe hacer el siguiente ajuste:
Fecha
07/10/20xx

Cdigo
2335
233523
5135
1110

Nombre de la Cuenta
Costos y Gastos por pagar
Electricaribe S.A,
Servicios
Bancos

Debe

Haber

287.200
12.800
300.000

Ejemplo: Si el dia 10 de Octubre se recibe la cuenta de cobro de Telmex S.A. por valor de $ 125.000, la
empresa Urbanizadora Vista al Mar debe hacer el siguiente ajuste:
Fecha
10/10/20xx

Cdigo
2335
233522
1110

Nombre de la Cuenta
Costos y Gastos por pagar
Telmex S.A,
Bancos

Debe

Haber

125.000
125.000

AJUSTE PARA DESGLOSAR EL COSTO CAUSADO


Una compra (o costo) que beneficiar a la empresa durante ms de un periodo contable, por ejemplo,
cuando una empresa adquiere un seguro, inicialmente debe considerar la totalidad del gasto pagado como
un activo e, ir reconociendo, ejercicio tras ejercicio, como gasto del ejercicio, nicamente la parte
proporcional de la adquisicin efectuada y, el saldo de dicho gasto continuara registrado en la cuenta de
activos diferidos hasta su consumo total.
Los activos diferidos comprenden los Gastos pagados por anticipado y los Cargos diferidos. Los Gastos
pagados por anticipado representan aquellos materiales que la empresa ha comprado para consumirlo en
un periodo futuro y los servicios pagados en la forma anticipada, contabilizados en la cuenta gastos
pagados por anticipado. Ejemplos: papelera, arrendamientos, impuestos, intereses, publicidad, seguros y
otros. Los Cargos diferidos, representan los costos y gastos en que incurre la empresa en las etapas de
organizacin, instalacin, montaje y puesta en marcha, adems de los gastos de inversin y estudio de
proyectos.
Ejemplo de la amortizacin de diferidos: Al llegar el extracto bancario, la empresa Pintura Arte y Decoracin
Ltda., apenas se entera que el 21 de Agosto de 20xx BANOCCIDENTE haba descontado directamente del
prstamo la suma de $ 225.000 por intereses anticipados de un trimestre. Calcular el ajuste de
amortizacin a 30 de junio.
Asiento de correccin
Fecha
31/08/20xx

Cdigo
1705
1110

Nombre de la Cuenta
Gastos pagados por anticipado
Bancos

Debe
225.000

Haber
225.000

Inters del trimestre $ 225.000, inters mensual $ 75.000


Tiempo de amortizacin:
Lapso entre el 21 y el 31 de agosto: 10 das
Valor de la amortizacin en 10 das: $ 25.000
Asiento de ajuste
Fecha
31/08/20xx

Cdigo
5305
1705

Nombre de la Cuenta
Financieros
Gastos pagados por anticipado

Debe

Haber
25.000
25.000

Para ver trabajos similares o recibir informacin semanal sobre nuevas publicaciones, visite www.monografias.com

www.monografias.com

Ejemplo de la depreciacin: En la apertura del negocio en ocho de agosto de 20xx la empresa Pinturas Arte
y Decoracin Ltda. contaba con un edificio por 220.000.000 de los cuales 95.000.000 corresponden al valor
del terreno. El valor de salvamento del edificio es de $ 5.000.000. Se pide el clculo de la depreciacin
anual, mensual y a 31 de Agosto, de cada bien depreciable.
Solucin: La depreciacin es el gasto en que incurre una empresa a medida que sus activos fijos tangibles
se desgastan durante la vida til. Puede estimarse que el activo se va consumiendo totalmente durante su
vida til o puede considerarse que al final conserva un valor residual, cuota de rescate o salvamento,
teniendo en cuenta el valor que tendr el activo al final de su vida til.
Vida til es el lapso durante el cual se espera que la propiedad, planta y equipo contribuyan a la generacin
de ingresos. Para su determinacin es necesario considerar una vida til tcnica fijada teniendo en cuenta
las especificaciones de fbrica, la obsolescencia por avances tecnolgicos, el desgaste debido al nmero de
unidades producidas, el deterioro por el tiempo, o a veces se toma como vida til legal aquella reglamentada
por el estatuto tributario.
El terreno no es un bien depreciable, mientras que la edificacin, la maquinaria, el equipo de oficina y los
vehculos si lo son. De Acuerdo con la legislacin tributaria colombiana, el tiempo de vida til y porcentaje de
desgaste establecido para efectos de la depreciacin anual, son los siguientes:
ACTIVO DEPRECIABLE
construcciones y edificaciones
maquinaria y equipo
equipo de oficina
equipo de computacin
flota y equipo de transporte

VIDA UTIL
2O aos
10 aos
10 aos
5 aos
5 aos

DEPRECIACION ANUAL (%)


5%
10%
10%
20%
20%

En este caso, la alcuota por depreciacin de la edificacin, se calcula as:


Valor que se registra en Edificio $ 220.000.000 - 95.000.000 5.000.000 = 120.000.000

Asiento de ajuste por la depreciacin del edificio:


Fecha
31/08/20xx

Cdigo
5160
51601604
1592

Nombre de la Cuenta
Depreciaciones
Bodega 250 mt2, Rebolo
Depreciacin Acumulada

Debe

Haber

383.333,00
383.333

Ejemplo del juego de Inventarios: El 31 de Agosto de 20xx, la empresa Pinturas Arte y Decoracin Ltda.,
realiza un inventario fsico de la mercanca y encuentra las siguientes existencias:

Para ver trabajos similares o recibir informacin semanal sobre nuevas publicaciones, visite www.monografias.com

www.monografias.com

El inventario es, por lo general, el activo mayor en los balances generales, y las erogaciones por inventarios,
llamadas Costo de Mercancas Vendidas, son usualmente el rubro mayor en el estado de resultados del
ejercicio. Existen dos sistemas de manejo del inventario: el sistema de inventario peridico y el sistema de
inventario permanente.
El sistema peridico es conocido tambin como sistema fsico, porque se apoya en el conteo fsico real del
inventario. En el sistema de inventario peridico el negocio registra las compras que se efectan durante el
presente periodo en la cuenta De Mercancas (como cuenta de costos). En esta cuenta De Mercancas no
se incluye la compra de Terrenos, Maquinarias, Edificios, Equipos, Instalaciones, etc. Esta cuenta tiene un
saldo deudor, no entra en el balance general de la empresa, y se cierra por Costo de Mercanca Vendida.
En el sistema peridico, las existencias de mercancas en inventario son registradas en la cuenta
Mercancas no Fabricadas por la Empresa. El valor inicial de esta cuenta, que hace parte del Balance
General del periodo anterior, sigue llevando el mismo valor durante todo el periodo. Sin embargo, al finalizar
el presente perodo, la cuenta Mercancas no Fabricada por la Empresa debe ser actualizada en los Estados
Financieros mediante un conteo fsico del inventario disponible a sus respectivos costos de adquisicin.
sta es la cifra que se utiliza para calcular el costo de las mercancas vendidas y que aparece como
Inventario en cada nuevo Balance General.
La tcnica del clculo del Costo de la Mercanca Vendida se conoce como Juego de Inventarios, la cual se
efecta del siguiente modo:
Saldo Inicial
+Compras (Nuevas adquisiciones. Bajo el sistema perpetuo significa los incrementos del inventario)
- Saldo Final (Conteo fsico del inventario disponible. Bajo el sistema perpetuo significa el costo del saldo
que arrojan las tarjetas de inventario en cualquier momento)
= Costo de la Mercanca Vendida
En el ejemplo que nos atae, podemos obtener el Costo de la Mercanca Vendida, as:

Inventario Inicial (aqu representado por el valor de $ 32.555.250) significa el valor de las existencias de
mercancas en la fecha que comenz el periodo contable. En la cuenta De Mercancas, se incluyen las
mercancas compradas durante el periodo contable con el objeto de venderlas. El Inventario Final (o actual,
aqu representado por el valor de $ 27.203.040) corresponde al inventario fsico de la mercanca realizado al
finalizar el periodo contable de la empresa con su correspondiente valoracin. Al relacionar la suma de estas
cuentas con el inventario final se obtendr el Costo de las Mercancas Vendidas.
As entonces, los asientos de ajuste por el juego de inventarios peridicos, son:
Fecha
31/08/20xx

Cdigo
1435
6205
1435
6135

Nombre de la Cuenta
Mercancas no Fab. por la emp.
De Mercancas
Mercancas no Fab. por la emp.
Comercio al P/M y al p/m

Debe

Haber
32.555.250
800.000

27.203.040
6.152.210

Para ver trabajos similares o recibir informacin semanal sobre nuevas publicaciones, visite www.monografias.com

www.monografias.com

En el sistema de Inventario Perpetuo, el negocio mantiene un registro continuo para cada artculo del
inventario. Los registros muestran por lo tanto el inventario disponible todo el tiempo. As, el negocio puede
determinar el costo del inventario final y el costo de las mercancas vendidas directamente de las cuentas
sin tener que hacer un conteo del inventario fsico ni tener que contabilizar nuevos valores del inventario.
Ejemplo: Suponiendo que la empresa Pintura Arte y Decoracin Ltda., haya adoptado el sistema de
inventario permanente y sabiendo que en 13 de septiembre de 20xx, vendi $ 2.700.000 en mercancas al
contado, cuyo costo dado por las tarjetas de inventario era de $ 1.620.000, se pide hacer los asientos
correspondientes:
Fecha
13/09/20xx

Cdigo
1105
4135
6135
1435

Nombre de la Cuenta
Caja
Comercio al P/M y al p/m
Comercio al P/M y al p/m
Mercanca no fab. por la empresa

Debe
2.700.000

Haber
2.700.000

1.620.000
1.620.000

Como veremos ms adelante, los registros perpetuos son tiles para preparar los estados financieros en
cualquier momento que sea necesario, ya sea diaria, mensual, trimestral o provisionalmente. En el sistema
de Inventario Permanente o perpetuo, al comparar el valor total del inventario fsico de mercancas, que
como mnimo debe efectuarse una vez por ao por exigencia de las autoridades tributarias, con el saldo en
libros, puede ocurrir que el inventario fsico resulte mayor que el valor en libros, en cuyo caso, debe
realizarse un ajuste por la diferencia del sobrante, debitando la cuenta de Mercancas no Fabricada por la
Empresa y acreditando la cuenta de Costo de la Mercanca Vendida. Generalmente esto ocurre por defectos
del registro en libros. De ocurrir lo contrario, se debita el Costo de la Mercanca Vendida y se acredita la
cuenta de Mercancas no fabricadas por la empresa, Por lo general se atribuyen los daos de la mercanca
o faltantes al manejo irresponsable de los inventarios
Al igual que cualquier otro ejercicio, se debe elaborar un resumen o Comprobante Diario de los asientos de
ajuste, as como el que se muestra a continuacin:

Para ver trabajos similares o recibir informacin semanal sobre nuevas publicaciones, visite www.monografias.com

www.monografias.com

3.3. Los Asientos de Cierre del Periodo Contable


Por regla general, el periodo contable es de un ao, comprendido entre el 01 de enero y el 31 de diciembre,
aunque en la prctica puede resultar conveniente trabajar con periodos de tiempo diferentes como el mes,
semestre, trimestre, etc.
Este principio supone que las operaciones econmicas, as como los efectos derivados de ellas, se
contabilizan de forma tal que se correspondan con el perodo econmico en que ocurren, para que las
informaciones contables muestren con claridad el perodo a que stas corresponden y pueda determinarse
el resultado de cada ejercicio econmico.
El cierre contable es el proceso consistente en cerrar o cancelar las cuentas de resultados y llevar su
resultado a las cuentas de balance respectivas.
Al finalizar un periodo contable, se debe proceder a cerrar las cuentas de resultado para determinar el
resultado econmico del ejercicio o del periodo que bien puede ser una prdida o una ganancia.
Recordemos que la ganancia o utilidad surge cuando los ingresos superan los costos y gastos, y la prdida
cuando los costos y gastos superan los ingresos. El resultado final de la cancelacin de las cuentas de
resultados, se debe llevar a la respectiva cuenta de patrimonio. Si el resultado es una prdida se disminuir
el patrimonio, y caso contrario, si el resultado es utilidad, la cuenta de patrimonio se incrementar.
ASIENTOS CONTABLES DE CIERRE
Estos asientos contables tienen como objetivo la cancelacin de las cuentas de resultado contra la cuenta
de prdidas y ganancias. Al finalizar el ao, y una vez se han realizado los ajustes pertinentes, se procede a
cancelar cada una de las cuentas de ingresos, gastos y costos para determinar la utilidad del ejercicio, y
esta cancelacin se registra en los comprobantes de cierre.
Es al momento del cierre contable cuando las cuentas de ingresos se debitan y las cuentas de gastos y
costos se acreditan contra la cuenta 5905 Ganancias y Prdidas, con el fin de cancelarlas, es decir, dejarlas

Para ver trabajos similares o recibir informacin semanal sobre nuevas publicaciones, visite www.monografias.com

www.monografias.com

en ceros. Estas cuentas para su cancelacin se debitan o acreditan por el mismo valor que tienen
acumulado.
Ejemplo: Sea la empresa Pintura Arte y Decoracin Ltda. cuyos ingresos estn constituidos por ventas de
mercancas y alguna utilidad en venta de propiedades. La eliminacin de estas cuentas procede de la
siguiente manera:
Fecha
31/08/20xx

Cdigo
4135
4245
5905

Nombre de la Cuenta
Comercio al P/M y al p/m
Utilidad en venta de prop, p y e
Ganancias y Prdidas

Debe
8.000.000
4.200.000

Haber

12.200.000

Por ser las cuentas de ingreso de saldo crdito, su cancelacin se efecta mediante anotaciones por su
mismo valor en el debito y utilizando como contracuenta la 5905 Ganancias y Perdidas
Del mismo modo, la cancelacin de los gastos y costos de la empresa Pinturas Arte y Decoracin se realiza
mediante la contracuenta 5905 Ganancias y Prdidas, as:

Fecha
31/08/20xx

Cdigo
5905
5120
5135
5160
5305
6135

Nombre de la Cuenta
Ganancias y Perdidas
Arrendamientos
Servicios
Depreciaciones
Financieros
Comercio al P/M y al p/m

Debe
7.760.543

Haber
860.000
340.000
383.333
25.000
6.152.210

Aqu vemos que la cuenta 5905 queda con un saldo crdito de $ 4.439.457 [esto es, Crdito: 12.200.000 Dbito: 7.760.543], saldo que tambin debemos cancelar contra la cuenta del patrimonio correspondiente a
resultados del ejercicio. El saldo de la cuenta 5905 nos indica que hay una utilidad, por lo tanto, la cuenta
5905 la cancelamos contra las cuentas 3605 Utilidad del Ejercicio, 3305 Reservas Obligatorias (10%) y 2615
Para Obligaciones Fiscales (35%):
Fecha
31/08/20xx

Cdigo
5905
3605
3305
2615

Nombre de la Cuenta
Ganancias y Perdidas
Utilidad del Ejercicio
Reservas Obligatorias
Para Obligaciones Fiscales

Debe
12.200.000

Haber
7.760.543
2.441.701
443.946
1.553.810

Al igual que cualquier otro ejercicio, se debe elaborar un resumen o Comprobante Diario de los asientos de
cierre del periodo, as:

Para ver trabajos similares o recibir informacin semanal sobre nuevas publicaciones, visite www.monografias.com

www.monografias.com

De esta forma se cancelan todas las cuentas de resultado, de modo que al hacer el cierre contable, slo nos
quedan las cuentas de balance activos, pasivos y patrimonio. Los valores contenidos en las cuentas de
resultado, al hacer el cierre se incorporan al patrimonio como utilidad o como prdida.
En conclusin, lo que se ha hecho es cancelar cada cuenta de resultado contra la cuenta 5905, y luego la
cuenta 5905 se cancela contra la cuenta 3605 o la 3610, segn sea el caso.
LOS PAPELES DE TRABAJO
Con el advenimiento de los recursos informticos al servicio del procesamiento de datos y clculos
numricos con diversas variables, en rapidez y volumen apreciable, la tecnologa contable continuar
progresando en su espectro de aplicaciones. (LOPES DE SA, 2008). Sin embargo, en contextos sociales de
escaso desarrollo tecnolgico, los medios mecnicos de forma columnaria como las llamadas hojas o
papeles de trabajo y los libros de contabilidad, seguirn en uso por mucho ms tiempo en numerosas
micros y pequeas empresas en la que logran ubicarse laboralmente muchos de nuestros contadores.
La forma columnaria, para organizar ordenadamente la informacin contable que se necesita para la
preparacin de los asientos de ajustes, los asientos de cierre y los estados peridicos, permite sumar
cmodamente los datos incluidos en los asientos de ajustes y cierre y, comprobar la igualdad de la suma de
saldos de los estados peridicos.
El procedimiento para preparar una Hoja de Trabajo es el siguiente:
1) Se escribe en el encabezamiento: el nombre de la empresa, el nombre del documento y la fecha de
elaboracin, en 3 lneas, centradas.
2) Se escriben los encabezamientos de las columnas, a saber: Balance de Comprobacin, Asientos de
Ajustes, Balance de Prueba Ajustado, Ingresos y Gastos, Balance General
3) Se anotan los datos del Balance de Comprobacin.
4) Se anotan los asientos de ajustes tomados del Comprobante Diario correspondiente. El concepto de
cada ajuste est indicado al pi de la hoja de trabajo, con una letra clave que aparece tambin junto
a los dbitos y crditos de las columnas de ajustes.
5) Se obtiene el Balance de Prueba Ajustado, sumando horizontalmente los movimientos
correspondientes de cada cuenta. Para obtener el resultado horizontalmente en cada cuenta de
Activos o de Gastos, se suman dbitos ms dbitos y se restan los crditos. En caso de ser cuentas
de Pasivos o de Ingresos, se suman crditos ms crditos y se restan los dbitos. De ser necesario,
se insertaran nuevas lneas para las cuentas necesarias teniendo que desplazar las cuentas del
Balance del Periodo anterior ya anotado en las dos primeras columnas. Se suman las columnas
Debe y Haber resultantes, las cuales deben arrojar sumas iguales.
6) Se anotan las partidas de ingresos y gastos a las columnas del Estado de ingresos. Se totalizar las
columnas del Estado de Ingreso.

Para ver trabajos similares o recibir informacin semanal sobre nuevas publicaciones, visite www.monografias.com

www.monografias.com

7) Se calcula y se anota el ingreso neto (o prdida neta).


8) Se lleva el ingreso neto al crdito de las columnas del Balance General.
9) Se trazan lneas dobles debajo de los ltimos totales de las columnas del Estado de Ingresos y
Balance General. Estas lneas muestran que todo el trabajo se concluy y se asume que este
correcto.
Las hojas de trabajo sirven como base para preparar:
El Estado de Resultado
El Balance General Clasificado
Los asientos de ajuste del diario
Los asientos de cierre del diario

3.4. Control de Inventarios


Las empresas dedicadas a la compra y venta de mercancas, por ser esta su principal funcin y la que dar
origen a todas las restantes operaciones, necesitaran de una constante informacin resumida y analizada
sobre sus inventarios, lo cual obliga a la apertura de una serie de cuentas principales y auxiliares
relacionadas con esos controles. Entres estas cuentas podemos nombrar las siguientes:

Para ver trabajos similares o recibir informacin semanal sobre nuevas publicaciones, visite www.monografias.com

www.monografias.com

Inventario (inicial)
Compras
Devoluciones en compra
Ventas
Devoluciones en ventas
Inventario (final)

Inventario inicial: Es la relacin detallada y minuciosa de las existencias de mercancas que tiene una
empresa al iniciar sus actividades, despus de hacer un conteo fsico.
Compras: En el sistema de inventario permanente no se utiliza la cuenta de compras (De Mercancas), sino
que las mercancas adquiridas entran directamente a la cuenta de inventarios (Mercancas no Fabricadas
por la Empresa). En ambos sistemas, el control interno de los inventarios se inicia con el establecimiento de
un departamento de compras, que deber gestionar las compras de los inventarios siguiendo el proceso de
compras. Los gastos ocasionados por las compras de mercancas, antes de llegar a su almacenamiento
dentro de la empresa, tales como fletes, recargos, etc., hacen parte integral del costo de adquisicin de la
mercanca.
Las Devoluciones en compra, reflejan toda aquella mercanca comprada que la empresa devuelve por
cualquier circunstancia; aunque esta cuenta disminuir la compra de mercancas no se abonar a la cuenta
compras.
Ventas: Esta cuenta controlar todas las ventas de mercancas realizadas por la Empresa y que fueron
compradas con este fin.
Devoluciones en Venta; la cual est creada para reflejar las devoluciones realizadas por los clientes a la
empresa.
Inventario final: Es la relacin de existencias al finalizar un periodo contable.
Los inventarios son contabilizados normalmente por su costo histrico, como lo requiere el principio del
costo. El costo del inventario es el precio que el negocio paga para adquirir el inventario, no el precio de
venta de las mercancas.
El costo de inventario incluye el precio de factura, menos cualquier descuento de compras, ms el
impuesto sobre las ventas, los aranceles, los cargos por transporte, el seguro mientras est en trnsito y
todos los otros costos en que se incurre para lograr que las mercancas estn disponibles para la venta.
EL SISTEMA DE INVENTARIO PERIODICO
Tal como ya ha sido mencionado anteriormente, en el sistema de inventario peridico, los comerciantes
determinan el valor de las existencias de mercancas mediante la realizacin de un conteo fsico en forma
peridica, el cual se denomina inventario inicial o final segn sea el caso.
El sistema peridico es generalmente utilizado para contabilizar los artculos del inventario que tienen un
costo unitario bajo. Para usar el sistema peridico con efectividad, el propietario debe tener la capacidad de
controlar el inventario mediante la inspeccin visual. Por ejemplo, cuando un cliente le solicita ciertas
cantidades disponibles, el dueo o administrador pueden visualizar las mercancas existentes para saber si
pueden atender tal pedido o no.
En el sistema peridico, el negocio registra las compras en la cuenta compras (como cuenta de gastos). Sin
embargo, al final del perodo, la cuenta inventario debe ser actualizada en los Estados Financieros mediante
conteo fsico. Estos asientos pueden realizarse como ajustes o en el proceso de cierre.
EL SISTEMA DE INVENTARIO PERMANENTE O PERPETUO
Las empresas que adoptan este tipo de sistema deben llevar un auxiliar de mercancas denominado
"krdex", en el cual se registra cada artculo que se compre o que se venda. La suma y la resta de todas las
operaciones en un periodo da como resultado el saldo final de mercancas.
Por medio de este sistema la empresa conoce el valor de la mercanca en existencia en cualquier momento,
sin necesidad de realizar un conteo fsico, por que los movimientos de compra y venta de mercancas se
registran directamente en el momento de realizar la transaccin a su precio de costo.
El sistema perpetuo ofrece un alto grado de control, porque los registros de inventario estn siempre
actualizados. Anteriormente, los negocios utilizaban el sistema perpetuo principalmente para los inventarios
de alto costo unitario, como las joyas y los automviles; hoy da con este mtodo los administradores
pueden tomar mejores decisiones acerca de las cantidades a comprar, los precios a pagar por el inventario,
la fijacin de precios al cliente y los trminos de venta a ofrecer. El conocimiento de la cantidad disponible
ayuda a proteger el inventario.

Para ver trabajos similares o recibir informacin semanal sobre nuevas publicaciones, visite www.monografias.com

www.monografias.com

En el sistema de inventario perpetuo, el negocio registra las compras de inventario cargando a la cuenta
inventario, cuando el negocio realiza una venta, se necesitan dos asientos. La compaa registra la venta de
la manera usual, carga a efectivo o a cuentas por cobrar y abona a ingresos por ventas el precio de las
mercancas vendidas. La compaa carga tambin a costo de mercancas vendidas y abona el costo a
inventario. El cargo a inventario (por las compras) sirve para llevar un registro actualizado del inventario
disponible. La cuenta inventario y la cuenta costo de mercancas vendidas llevan un saldo actual durante el
periodo.
3.5. Mtodos para la fijacin del costo de la mercanca
Los mtodos de costeo de inventarios son: costo de primeras en entrar primeras en salir (PEPS), costo de
ltimas en entrar primeras en salir (UEPS) y costo del promedio ponderado.
COSTO DE PRIMERAS EN ENTRAR, PRIMERAS EN SALIR (PEPS)
Bajo el mtodo de primeras entradas, primeras salidas, la compaa debe llevar un registro del costo de
cada unidad comprada del inventario. El costo de la unidad utilizado para calcular el inventario final, puede
ser diferente de los costos unitarios utilizados para calcular el costo de las mercancas vendidas. Bajo
PEPS, los primeros costos que entran al inventario son los primeros costos que salen al costo de las
mercancas vendidas, a eso se debe el nombre de Primeras Entradas, Primeras Salidas. El inventario final
se basa en los costos de las compras ms recientes.
Ejemplo: Sea una empresa con un inventario inicial de 15 unidades con valor unitario de $ 20.0000, luego
efecta compras por 25 unidades a valor unitario de $ 30.000. Posteriormente obtiene pedidos por 40
unidades. Se pide fijar el costo de la mercanca a vender y el precio de venta si se desea lograr un margen
de utilidad del 60%. Explicar la aplicacin de cada uno de los mtodos para la fijacin del costo de
mercancas en el inventario.
Concepto
Inventario Inicial
Compras
Total disponible para las ventas
Ventas
Inventario Final

Cantidad
15
25
40
35
5

Vr. Unitario
$ 20.000
30.000

Costo
$ 300.000
750.000
1.050.000

El Costo de la Mercanca Vendida se obtiene mediante la aplicacin del Juego de Inventarios que muestra
el cuadro anterior. Al vender 35 unidades bajo el mtodo PEPS, ha de tomar las 15 del inventario inicial ms
20 tomadas de la primera compra, esto es:
15 Unds. x $ 20.000 c/u + 20 Unds. x $ 30.000 c/u = $ 900.000.
De este modo en el inventario final solo quedaran 5 unidades valoradas al costo de la ltima compra:
Valor del inventario final = 5 Unid. x $ 30.000 = $ 150.000
El Precio de Venta se calcula aplicando la frmula del margen de utilidad:
Precio de Venta del lote = Costo / (1-margen) = 900.000 / 0,40 = $ 2.250.000
COSTO DE ULTIMAS EN ENTRAR, PRIMERAS EN SALIR (UEPS)
El mtodo ltimas entradas, primeras salidas dependen tambin de los costos por compras de un inventario
en particular. Bajo este mtodo, los ltimos costos que entran al inventario son los primeros costos que
salen al costo de mercancas vendidas. Este mtodo deja los costos ms antiguos (aquellos del inventario
inicial y las compras primeras del periodo) en el inventario final.
Aqu se toman 25 unidades de la ltima compra y tan solo 10 del inventario inicial. Aplicando la misma lgica
del mtodo anterior, tendremos:
Unidades Vendidas: 25 Unds. x $ 30.000 c/u + 10 Unds. x $ 20.000 c/u = $ 950.000.
El inventario final queda valorado al costo que tenia la mercanca en el inventario inicial.
Valor del inventario final = 5 Unid. x $20.000 = $ 100.000
Precio de Venta = 950.000 / 0,40 = $ 2.375.000
Las principales ventajas del sistema de inventario permanente se notan en la oportunidad de la informacin
las siguientes decisiones:
1. En la mayora de los negocios comerciales, el grueso de la mercanca es guardada en secciones
diferentes a los almacenes, por lo tanto los empleados no pueden examinar visualmente la

Para ver trabajos similares o recibir informacin semanal sobre nuevas publicaciones, visite www.monografias.com

www.monografias.com

mercanca disponible y dar respuesta en ese mismo instante. El sistema perpetuo le indicar
oportunamente la disponibilidad de dicha mercanca.
2. Los sistemas del registro perpetuo son capaces de alertan al negocio sobre el bajo nivel de las
existencias.
3. Con los registros de inventario perpetuo, no es necesario un conteo fsico a cada momento; sino
que ello solo se hace una vez al ao para verificar la exactitud de los registros.
COSTO DEL PROMEDIO PONDERADO
El mtodo del costo promedio ponderado, llamado a menudo mtodo del costo promedio se basa en el
costo promedio ponderado del inventario durante el perodo. Este mtodo pondera el costo por unidad como
el costo unitario promedio durante un periodo, esto es, si el costo de la unidad baja o sube durante el
periodo, se utiliza el promedio de estos costos.
El costo promedio se determina de la manera siguiente: divida el costo de las mercancas disponibles para
la venta (inventario inicial + compras) entre el nmero de unidades disponibles.

Calcule el inventario final y el costo de mercancas vendidas, multiplicando el nmero de unidades por el
costo promedio por unidad.
El valor promedio del costo por artculo resulta ser de $ 26.250
Unidades Vendidas: 35 Unds. x $ 26.250 c/u = $ 918.750.
El inventario final queda valorado al costo promedio mercanca en existencia, esto es:
Valor del inventario final = 5 Unid. x $26.250 = $ 131.250
Precio de Venta = 918.750 / 0,40 = $ 2.296.875
Al analizar los tres mtodos se puede sacar como conclusin que la valoracin ms baja es la obtenida con
el PEPS, la ms alta con el UEPS y una valoracin intermedia con el promedio.
Ejemplo de Inventario Permanente: Si en determinada empresa, el costo de las mercancas disponibles para
la venta es de $90,000 y estn disponible 60 unidades, el costo del promedio es de $1,500 por unidad. El
inventario final de 20 unidades del mismo artculo tiene un costo promedio de $30,000 (20 x $1,500 = $
30,000). El costo de las mercancas vendidas (40 unidades) es de $60,000 (40 x $1,500 = $60,000).
Sabiendo que el margen de utilidad sobre ventas es del 25% y que la empresa otorga un descuento del 10%
por ventas al contado, hacer el asiento de dicha venta.
Solucin: Se requiere conocer el precio de venta, as:
Precio Unitario de Venta = Costo / (1-margen) = 1.500 / 0,75 = $ 2.000 / und.
El valor de la venta ha sido entonces. 40 unidades x $ 2.000 / und. = $ 80.000
El asiento para registrar la venta con descuento al contado, es:

Fecha
05/07/20xx

Cdigo
1105
5305
530535
6135
1435
4135

Nombre de la Cuenta
Caja
Financieros
Descuentos comerciales condic.
Comercio al P/M y p/m
Mercancas no fabricadas p/e
Comercio al P/M y p/m

Debe

Haber
72.000
8.000
60.000
60.000
80.000

Otro Ejemplo de Inventario Promedio: El 31 de Enero de 20xx, la empresa Atomizadores del Norte S.A.,
tena la existencia de 300 unidades de atomizadores de 300 ml., a un costo de adquisicin de $ 10.000 cada
unidad. En 2 de Febrero la empresa adquiere otras 200 unidades a un costo de $ 12.500 por unidad, las
cuales pago con un cheque del Banco de Bogot.. Suponiendo que la empresa utiliza un sistema de
inventario permanente bajo mtodo del promedio ponderado, confeccionar los asientos de compra y de la

Para ver trabajos similares o recibir informacin semanal sobre nuevas publicaciones, visite www.monografias.com

www.monografias.com

venta tambin al contado del 9 de febrero de 400 unidades suponiendo un margen de utilidad sobre el
precio unitario de venta del 45%
Solucin: El asiento de la compra se efecta en los siguientes trminos:

Fecha
02/02/20xx

Cdigo
1435
143566
1110
111005
11100502

Nombre de la Cuenta
Mercancas no Fab. p/Empresa
Empaques atomizadores 300 ml
Bancos
Nacionales
Banco de Bogot

Debe

Haber

2.500.000

2500.000

Para vender 400 unidades bajo el mtodo del Promedio Ponderado, se requiere calcular el costo promedio
de las existencias, as:
Inventario actual: 300unidades x $ 10M + 200 unidades x $ 12,5M = $ 5.500 M

Precio Unitario de Venta = Costo / (1-margen) = 11.000 / 0,55 = $ 20.000 / und.


El asiento para registrar la venta a crdito, es:
Fecha
05/07/20xx

Cdigo
1105
6135
613566
1435
143566
4135
413566

Nombre de la Cuenta
Caja
Comercio al P/M y p/m
Venta de Empaques
Mercancas no fabricadas p/e
Empaques atomizadores 300 ml
Comercio al P/M y p/m
Venta de Empaques

Debe

Haber
8.000.000
4.400.000
4.400.000
8.000.000

3.6. Evaluacin
3.6.1. Problemas Resueltos
1- La empresa la Poderosa S.A. cierra sus libros mensualmente, paga en 1 de Febrero la suma de
$ 40.000.000 a Telecaribe por una campaa publicitaria que tiene una duracin de 8 meses, se
requiere:
a) El asiento para registrar el pago en 1 de febrero
b) El asiento de ajuste requerido al respecto en mayo
c) Por cuanto aparecer la cuenta de Gastos pagados por anticipado en el balance general a
31 de mayo?
d) Si la empresa la Poderosa S.A. hubiera cargado todos los $ 40.000.000 a Gastos en
Publicidad del primer mes, cul sera el efecto de ello sobre la ganancia neta en cada uno
de los ocho meses siguientes comenzando en 1 de febrero?
Solucin: Este es un tpico ejercicio de ajuste para desglosar el costo causado, mes a mes, as:
a) El asiento para registrar el pago anticipado es:
Fecha
01/02/20xx

Cdigo
1705
170540
17054060

Nombre de la Cuenta
Gastos pagados por anticipado
Servicios
Publicidad y promocin

Debe

Haber

40.000.000

Para ver trabajos similares o recibir informacin semanal sobre nuevas publicaciones, visite www.monografias.com

www.monografias.com

1110
111005
1110502

Bancos
Moneda Nacional
Banco de Bogot

40.000.000

b) Antes de la confeccin del asiento de ajuste, se requiere el clculo de la alcuota de costo,


as:

Fecha
01/02/20xx

Cdigo
5235
523560
1705
170540
17054060

Nombre de la Cuenta
Servicios
Publicidad, Propaganda y Prom.
Gastos pagados por anticipado
Servicios
Publicidad y promocin

Debe

Haber
5.000.000

5.000.000

c) Al mes de mayo, han transcurrido 4 meses, por lo tanto se tiene:


Costo causado: 4 meses x $ 5.0000 / mes = $ 20.000.000
Costo por causar: $ 40.000.000 - $ 20.000.000 = $ 20.000.000
d) Si la empresa hubiese cargado la totalidad de la suma pagada durante el primer mes,
estara afectando la ganancia del primer mes por un costo que no le corresponde a ella
sola, mientras que por el contrario, la ganancia de los dems meses se vera favorecida
(inflada) debido a no tener que sufragar un costo que en realidad es de su causalidad.

2- La Cooperativa Wayunaki posea un inventario a 31 de diciembre del ao anterior avaluado en $


164.000.000. Sus compras durante el ao ascendieron a $ 493.000.000. Si su inventario a 31 de
diciembre del presente ao fue avaluado en $ 201.000.000, cual es el costo de la mercanca
vendida?. A cmo debieron venderse tales 5.700 mochilas guajiras para obtener una ganancia
bruta sobre las ventas netas del 36%?
Solucin: Este es un ejercicio de ajuste llamado Juego de Inventarios, el cual requiere
previamente el clculo del costo de la mercanca vendida, as:

Los asientos de ajuste por el juego de inventarios, son:


Fecha
31/08/20xx

Cdigo
1435
6205
1435
6135

Nombre de la Cuenta
Mercancas no Fab. por la emp.
De Mercancas
Mercancas no Fab. por la emp.
Comercio al P/M y al p/m

Debe

Haber
164.000.000
493.000.000

201.000.000
456.000.000

Para ver trabajos similares o recibir informacin semanal sobre nuevas publicaciones, visite www.monografias.com

www.monografias.com

Precio Unitario de Venta = Costo / (1-margen) = 80.000 / 0,64 = $ 125.000 / und.


3- El Balance de Comprobacin de la Ca. Ganimedes Ltda. realizado el da Viernes 28 de Julio,
muestra los siguientes guarismos:

El Informe de los Gastos Causados incluyendo el da feriado Lunes 31 de Julio consiste de:
a) Ingresos por Servicios de Entretenimiento Prestados aun sin cobrar : $ 800.000
b) Intereses devengados por la Cdula de Capitalizacin: $200.000
c) Intereses acumulados sobre los Papeles Comerciales: $ 300.000
d) Salarios Causados: $ 1.200.000
e) Impuesto predial causado sobre el Terreno: $ 40.000
f) Se decretaron las Participaciones Sociales del mes: $ 600.000
Se requiere: Asientos de Ajuste en el Diario y su Comprobante de Diario, Balance de Prueba
Ajustado, Asientos de Cierre en el Diario y su Comprobante de Diario, el Estado de Ingresos y
Gastos y el Balance General.
Solucin: Este es un ejercicio completo de ajuste y cierre, el cual se convierte en un proceso
repetitivo cada final de periodo:
a) El asiento para registrar el ingreso obtenido durante el da feriado es:
Fecha
29/07/20xx

Cdigo
1345
134525
13452540
4170
417040

Nombre de la Cuenta
Ingresos por cobrar
Servicios
Entretenimiento y Esparcimiento
Actividades de Serv. Comunitario
Entretenimiento y Esparcimiento

Debe

Haber

800.000
800.000

b) El asiento para registrar los intereses generados por la Cedula, es:

Para ver trabajos similares o recibir informacin semanal sobre nuevas publicaciones, visite www.monografias.com

www.monografias.com

Fecha
29/07/20xx

Cdigo
1345
134510
13451002
4210
421005

Nombre de la Cuenta
Ingresos por cobrar
Intereses
Banco de Bogot
Financieros
Intereses

Debe

Haber

200.000
200.000

c) El asiento para registrar los intereses adeudados por la financiacion, es:


Fecha
30/07/20xx

Cdigo
5305
530520
2335
233505
23350501

Nombre de la Cuenta
Financieros
Intereses
Costos y Gastos por Pagar
Gastos financieros
Banco de Crdito

Debe

Haber
300.000

300.000

d) El asiento para registrar la causacin de los salarios, es:


Fecha
30/07/20xx

Cdigo
5105
510512
2505
250505

Nombre de la Cuenta
Gastos de Personal
Jornales
Salarios por Pagar
Pedro Yudex

Debe

Haber
1.200.000
1.200.000

e) El asiento para registrar la causacin del impuesto predial, es:


Fecha
31/07/20xx

Cdigo
5215
521515
2416
241601
f)

Fecha
31/07/20xx

Nombre de la Cuenta
Impuestos
Gasto de impuesto a la prop. raz
Otros impuestos
Impuesto a la propiedad raz

Debe

Haber
40.000
40.000

El asiento para registrar la causacin de la distribucin de utilidades, es:


Cdigo
3705
370505
2195
219520

Nombre de la Cuenta
Utilidades o excedentes acum.
Utilidades por repartir
Otras Obligaciones
Con Socios y Accionistas

Debe

Haber
600.000
600.000

El Comprobante de Diario de los asientos de ajuste es como sigue:

Para ver trabajos similares o recibir informacin semanal sobre nuevas publicaciones, visite www.monografias.com

www.monografias.com

Los ingresos de la compaa Ganimedes estn constituidos por ventas de servicios. Por ser las cuentas de
ingreso de saldo crdito, su cancelacin se efecta mediante anotaciones por su mismo valor en el debito y
utilizando como contracuenta la 5905 Ganancias y Prdidas. La eliminacin de estas cuentas procede de la
siguiente manera:
Fecha
31/07/20xx

Cdigo
4170
4210
5905

Nombre de la Cuenta
Actividades de serv. comunitario
Financieros
Ganancias y Prdidas

Debe
17.800.000
200.000

Haber

18.000.000

Del mismo modo, la cancelacin de los gastos y costos de la compaa Ganimedes se realiza mediante la
contracuenta 5905 Ganancias y Prdidas, as:
Fecha
31/07/20xx

Cdigo
5905
5105
5120
5135
5215
5235
5305

Nombre de la Cuenta
Ganancias y Perdidas
Gastos de Personal
Arrendamientos
Servicios
Impuestos
Servicios
Financieros

Debe
17.240.000

Haber
13.200.000
1.000.000
700.000
40.000
2.000.000
300.000

La cuenta 5905 queda con un saldo crdito de $ 560.000 [esto es, Crdito: 17.800.000 - Dbito:
17.240.240], el cual corresponde a una utilidad, por lo tanto, la cuenta 5905 la cancelamos contra las
cuentas 3605 Utilidad del Ejercicio, 3305 Reservas Obligatorias (10%) y 2615 Para Obligaciones Fiscales
(35%):
Fecha
31/07/20xx

Cdigo
5905
3605

Nombre de la Cuenta
Ganancias y Perdidas
Utilidad del Ejercicio

Debe
18.000.000

Haber
17.240.000
418.000

Para ver trabajos similares o recibir informacin semanal sobre nuevas publicaciones, visite www.monografias.com

www.monografias.com

3305
2615

Reservas Obligatorias
Para Obligaciones Fiscales

76.000
266.000

El Comprobante de Diario de los asientos de cierre nos permitir trasladar estos movimientos de las cuentas
temporales a los Papeles de Trabajo y al Libro Mayor:

Para ver trabajos similares o recibir informacin semanal sobre nuevas publicaciones, visite www.monografias.com

www.monografias.com

4- En el inventario de Marzo 1 de la empresa Drogas Apacibles S.A., se encontraron 70 unidades


de inyecciones analgsicas para enfermos terminales con un valor total de $ 2.800.000. Las
compras de este medicamento fueron durante el mes las siguientes:
a) 06.03._ 60 unidades a $ 41.000 c/u
b) 14.03._ 80 unidades a 41.200 c/u
c) 23.03._ 40 unidades a $ 41.400 c/u
d) 28.03._ 60 unidades a 41.300 c/u
Las ventas del medicamento fueron las siguientes:
A) 03.03._ 20 unidades
B) 10.03._ 40 unidades
C) 15.03._ 30 unidades
D) 24.03._ 80 unidades
E) 29.03._ 70 unidades
Se pide:
Prepare la tarjeta de existencias para el medicamento, aplicando el mtodo PEPS de valuacin
de inventarios.
Solucin: En los ejercicios con tarjeta de inventario, las compras no ofrece ninguna dificultad
pues siempre se conoce el precio de adquisicin de las mercancas. El problema lo plantean las
requisiciones de mercancas del almacn, las cuales requieren del clculo del costo a la que
son retiradas:

Para ver trabajos similares o recibir informacin semanal sobre nuevas publicaciones, visite www.monografias.com

www.monografias.com

Tomando como ejemplo, el retiro de mercancas del 15/03 mostrada en el siguiente cuadro
tendremos:
En esa fecha se tena la existencia de 150 unidades las cuales consistan en: 10 unidades x $
40.000 + 60 unidades x $ 41.000 + 80 unidades x $ 41.200 = $ 6.156.000
Para vender 30 unidades bajo el mtodo PEPS, se requiere tomar:
10 unidades x $ 40.000 + 20 unidades x $ 41.000 = $ 1.220.000
De este modo en el inventario final solo quedaran 120 unidades valoradas, as:
40 unidades x $ 41.000 + 80 unidades x $ 41.200 = $ 4.936.000
5- La empresa Buses y Camiones S:A., inicia operaciones con 300 chasises de 30 toneladas, cada
uno con un costo de $ 40.000.000. Las importaciones que hizo de este tipo de chasises durante
el mes de Julio fueron:
a) 03.07._ 600 unidades a $ 43.000.000
b) 14.07._ 600 unidades a $ 43.800.000
c) 22.07._ 800 unidades a $ 45.250.000
Las ventas de estos chasises fueron:
a) 05.07._ 700 unidades
b) 18.07._ 700 unidades
c) 21.07._ 50 unidades
d) 28.07._ 560 unidades
Se pide:
1) Preparar la tarjeta de existencias de los chasises suponiendo que la empresa utiliza un
sistema de inventario permanente bajo mtodo UEPS.
2) Confeccionar el asiento de la venta a crdito del 5 de Julio suponiendo un margen de
utilidad sobre el precio unitario de venta del 63%
Solucin: La tarjeta del movimiento de las mercancas se muestra en el siguiente cuadro:

Para ver trabajos similares o recibir informacin semanal sobre nuevas publicaciones, visite www.monografias.com

www.monografias.com

En 05/07 se tena la existencia de 900 unidades las cuales consistan en:


300 unidades x $ 40MM + 600 unidades x $ 43MM = $ 37.800 MM
Para vender 700 unidades bajo el mtodo ueps, se requiere calcular el costo de las existencias
retiradas, as:
600 unidades x $ 43MM + 100 unidades x $ 40MM = $ 29.800 MM

Precio Unitario de Venta = Costo / (1-margen) = 42,57 MM / 0,37 = $ 115 MM / und.


El asiento para registrar la venta a crdito, es:
Fecha
05/07/20xx

Cdigo
1305
130505
13050533
6135
613502
1435
143567
4135
413502

Nombre de la Cuenta
Clientes
Nacionales
N.N.
Comercio al P/M y p/m
Venta de Vehculos Automotores
Mercancas no fabricadas p/e
Chasises para colectivos
Comercio al P/M y p/m
Venta de Vehculos Automotores

Debe

Haber

80.540.540.540
29.800.000.000
29.800.000.000
80.540.540.540

3.6.2. Problemas Propuestos


1- La empresa Urbanizadora Anobia Ltda.,, dedicada a la construccin de vivienda social, recibe
un avance de $ 300.000.000 en 6 de mayo. Esta empresa cierra sus libros cada trimestre (mayo
31, Junio 30, etc...) y en 31 de mayo estima que sus gastos en obras hasta esa fecha ascienden
a $ 60.000.000. El contrato termin en Junio 31, fecha en la que el Contratante acept cargar
otros $ 90.000.000, es decir que la obra cuesta en total $ 390.000.000. Se requiere:
a) El asiento para los $ 300.000.000 recibidos como avance en 6 de mayo
b) Los asientos requeridos en mayo 31 y junio 30
c) Todos los asientos requeridos al final del contrato.
2- Elabore los siguientes asientos de ajuste necesarios en 31 de diciembre prximo, teniendo en
cuenta que los libros se cierran solo una vez al ao:
a) En 1 de octubre del mismo ao, se compro una pliza de seguro a 3 aos por la suma de $
3.000.000 y se carg a Gastos pagados por anticipado

Para ver trabajos similares o recibir informacin semanal sobre nuevas publicaciones, visite www.monografias.com

www.monografias.com

b) Los materiales de consumo comprados dentro del ao costaron $ 9.400.000, los cuales se
cargaron a Materiales, Repuestos y Accesorios. Esta cuenta tena un saldo de $ 2.600.000
en 1 de Enero y al tomar el inventario en 31 de diciembre arroj un valor de $ 2.000.000
c) En 1 de Noviembre se pago por anticipado el alquiler de 6 meses por un valor de $
5.400.000, lo cual se cargo a Gastos pagados por anticipado
d) Hasta 31 de diciembre se han devengado $ 230.000 por concepto de intereses sobre una
Letra por Cobrar, lo cual aun no ha sido cobrado
e) En 31 de diciembre se deben salarios acumulados por $ 17.000.000
f) Del total de $ 39.000.000 acreditados a Ingresos recibidos por anticipado, durante el ao, $
10.000.000 fueron devengados hasta 31 de diciembre
g) La empresa deprecia sus edificios en 20 aos. El saldo de la cuenta de Construcciones y
edificios en 1 de enero era de $ 408.000 y el de la Depreciacin acumulada de edificios de $
72.000.000. El 1 de julio la empresa adquiere otro edificio por un costo total de $
140.000.000 de los cuales $ 20.000.000 se asignaron a la cuenta de Terrenos.
3- A continuacin aparece el Balance de Comprobacin de la Compaa Manos a la Obra S.A.,
correspondiente al 31 de Diciembre de 2006. Esta Compaa cierra sus libros cada ao en esta
misma fecha:

Se requiere: Asientos de Ajuste, Comprobante de Diario y Balance de Prueba ajustado teniendo en


cuenta los siguientes ajustes:
a) Salarios Acumulados a 31/12/xx: $ 1.000.000
b) Inters devengado y no cobrado sobre Letras por Cobrar $ 150.000
c) En 31/12/xx haba Materiales en existencia por valor de $ 6.250.000
d) El edificio tiene una vida til estimada de 25 aos
e) Se recibi un depsito de $ 3.000.000 de la Compaa Quintal S.A. para servicios futuros,
cuya suma fue registrada como ingresos por servicios. En 31/12/xx an no se haba
comenzado este trabajo.
4- Usando los datos que aparecen abajo, se pide hacer los Asientos de Cierre, Comprobante de
Diario, Estado de Ingresos y Gastos y Balance General:

Para ver trabajos similares o recibir informacin semanal sobre nuevas publicaciones, visite www.monografias.com

www.monografias.com

Sabiendo QUE:
a) Los INGRESOS durante el mes fueron de $ 52.000.000
b) Los EGRESOS fueron: - Alquiler $ 8.000.000, - Salarios: $ 20.000.000, - Gastos diversos: $
13.000.000. Adems, se repartieron $ 4.000.000 en dividendos
5- Con la siguiente informacin prepare los papeles de trabajo para la Ca GIOVANNA S.A. del
mes de Enero de 20xx:

Informacin adicional:
a) Inters acumulado sobre Letras por Cobrar. $ 100.000
b) Materiales utilizados en Enero: $ 2.000.000
c) Depreciacin de Equipo de Oficina: 20% por ao
d) Inters Acumulado sobre Letras por Pagar: $ 200.000
e) Salarios Acumulados por Pagar: $ 4.000.000
f) Impuesto Predial Acumulado por Pagar: $ 1.000.000
g) Alquiler Pagado por Anticipado en 31 de Enero: $ 2.000.000
h) El seguro se compr en Octubre 1 del ao anterior por dos aos
i) El Inventario de Mercancas en 31 de Enero vala $ 70.000.000
6- A partir del Balance de Comprobacin obtenido para la Lavandera Automtica del problema 17
del captulo anterior y los siguientes datos de ajuste construir un Balance de Comprobacin
ajustado, los asientos de cierre necesarios en el Diario y el Mayor, el Estado de Ingresos y

Para ver trabajos similares o recibir informacin semanal sobre nuevas publicaciones, visite www.monografias.com

www.monografias.com

Egresos y el Balance General, teniendo en cuenta que los libros de la compaa se cierran cada
mes:
a) En 15 de Enero se compr una pliza de seguros a 3 aos por la suma de $ 7.500.000,
b) Al hacer un inventario se encontr que haban sobrado $ 25.000 en Materiales de consumo
de la lavandera.
c) El 31 de Enero pag $ 4.500.000 por pago adelantado de 3 meses de arrendamiento.
d) Hasta 31 de Enero se han causado $ 180.000 por concepto de intereses sobre la deuda
comercial sin haberlos recibido todava.
e) Del total de los $ 6.100.000 acreditados en Cuentas Corrientes Comerciales el da 31 del
mes fueron entregados $ 1.000.000,
f) La Compaa deprecia los Equipos de Oficina a 10 aos y los vehculos a 5 aos.
7- A partir del Balance de Comprobacin del mes de Junio obtenido para la Compaa Picapiedra
S.A. en el problema 19 del captulo anterior y los siguientes datos de ajuste construir el prximo
informe semestral que deber remitir a la casa principal:
Los asientos de ajuste para el ltimo bimestre han de ser:
a) En el mes de Junio expiraron $ 375.000 de Gastos de Seguros pagados por anticipado
b) Deveng renta adicional por intereses sobre inversiones en certificados a trmino
c) El camin de reparto se deprecia en 5 aos
d) La Provisin del Impuesto de Rodamiento del bimestre fue de $ 18.500
e) En 30 de Junio se haba consumido el 10% de los tiles de oficina
f) El valor del Inventario de Mercancas en existencias al 30 de Junio fue de $ 31.000.000
Hacer los ajustes necesarios en el Diario y confeccionar el Comprobante Diario de estos ajustes, el
Balance de Comprobacin ajustado, los asientos de cierre necesarios en el Diario y el
correspondiente Comprobante de Diario, el Estado de Ingresos y Egresos y el Balance General
8- A continuacin aparece el Balance de Comprobacin de la Ca. Jpiter S:A: correspondiente a
31 de Diciembre de 20xx. Esta Compaa cierra sus libros una vez al ao en esta fecha:

Para ver trabajos similares o recibir informacin semanal sobre nuevas publicaciones, visite www.monografias.com

www.monografias.com

DATOS PARA LOS AJUSTES:


a) Salarios Acumulados por Pagar en 31 de Diciembre de 20xx: $ 5.500.000
b) Los edificios se deprecian a 20 aos y la maquinaria a 10 aos. No se compraron edificios ni
maquinarias durante el ao.
c) El Inters acumulado sobre las Letras por Pagar a 31 de Diciembre/xx es de $ 10.000.000
d) El Inters acumulado sobre las Letras por Cobrar es de $ 8.000.000
e) En 31 de Diciembre an no se haban cancelado $ 3.000.000 del saldo de la cuenta de Ingresos
por Servicios Recibidos por Anticipado
f) El Valor del Seguro Pagado por Anticipado que aparece en el Balance de Comprobacin
corresponde a una pliza de seguro contra incendio a 3 aos, cuya vigencia se cuenta a partir
del 1 de Abril de 20xx.
g) Se decretaron $ 10.000.000 de Dividendos
Hacer los ajustes necesarios en el Diario y su Comprobante de Diario, el Balance de Comprobacin
ajustado, los asientos de cierre necesarios en el Diario y su Comprobante de Diario, el Estado de
Ingresos y Egresos y el Balance General
Cul habra sido el asiento de Depreciacin para la maquinaria en 31 de Diciembre si los $
120.000.000 que aparecen en el Balance de Comprobacin provinieran de:
Saldo a 01/01/99: $ 90.000.000,
Compra de Maquinaria en 11/07/99: $ 30.000.000?
- Cul habra sido el asiento para el gasto de seguro si la suma que aparece en el Balance de
Comprobacin hubiese sido una pliza a cinco aos comprada en 1 de Mayo/05?
9- Las cuentas que aparecen abajo estn en el libro Mayor de la Compaa Delantal S.A., en 30 de
Abril de 20xx:

Para ver trabajos similares o recibir informacin semanal sobre nuevas publicaciones, visite www.monografias.com

www.monografias.com

Durante el mes de Mayo la Compaa Delantal S.A. tuvo las siguientes transacciones:
a) En mayo 1 Compra de Mercancas a Crdito por $ 190.000.000
b) En Mayo 1 Compra de Muebles de Oficinas al Contado por $ 1.200.000
c) En mayo 3 Venta de Mercancas a Crdito por $ 92.000.000
d) En mayo 6 Venta de Mercancas a Crdito por $ 71.000.000
e) En mayo 7 Cobro a Clientes por $ 125.000.000
f) En mayo 10 Cobro a Clientes por $ 129.000.000
g) En mayo 11 Pago a Proveedores por $ 136.000.000
h) En mayo 12 Venta de Mercancas al Contado por $ 62.000.000
i) En mayo 14 Compra de Mercancas al Contado por $ 73.000.000
j) En mayo 15 Pago de la 1. Quincena a trabajadores por $ 90.000.000
k) En mayo 20 Propaganda del Heraldo por $ 5.000.000
l) En mayo 22 Gastos de Correos por $ 1.000.000
m) En mayo 23 Ventas a Crdito por $ 169.000.000
n) En mayo 26 Ventas a Crdito por $ 171.000.000
o) En mayo 27 Cobro a Clientes por $ 362.000.000
p) En mayo 31 Pago de la 2. Quincena por $ 102.000.000
q) En mayo 31 Pago a Proveedores por $ 54.000.000
Datos para los ajustes:
A) Depreciacin de Edificios $ 7.000.000/mes.
B) Depreciacin de Equipos de Oficinas $ 5.000.000/mes.
C) Impuesto predial causado en Mayo $ 1.000.000
D) Gasto de Seguros causado en Mayo $ 2.000.000
E) Inventario de mercancas en existencias en 31 de Mayo. $ 195.000.000
Se requiere:
1) Hacer asientos de diario, su Comprobante de Diario y Balance de Comprobacin,
2) Hacer asientos de diario de ajuste y de cierre a 31 de Mayo con sus respectivos Comprobantes
de Diario,
3) Preparar el Estado de Ingresos y Gastos y el Balance General.
4) Preparar los papeles de trabajo del mes de Mayo,

Para ver trabajos similares o recibir informacin semanal sobre nuevas publicaciones, visite www.monografias.com

www.monografias.com

10- Con la siguiente informacin calcule el costo del inventario a 31 de diciembre:


a) Inventario Inicial de mercancas a 1 de Enero: $ 90.440.000
b) Compras del ao: $ 308.020.000
c) Fletes de entrada. $ 12.000.000
d) Devoluciones en compras: $ 22.060.000
e) Ventas del ao: $ 454.400.000 con ganancia bruta sobre ventas del 37,5%
11- Registre por los sistemas de inventario peridico las siguientes operaciones realizadas por la
empresa Comercial Andina, de Cali.
a) Compra mercancas al contado a Mercosur Ltda. de Cali por $3,850,000; se paga con
cheque.
b) Devuelve 20% de la mercanca comprada en el literal a.
c) Vende mercancas a crdito a Margarita Mariel, del rgimen simplificado, por $1.650.000.
d) La cliente del literal c paga anticipadamente su cuenta y le conceden 3% de descuento por
pronto pago sobre el valor de la mercanca.
e) Compra mercanca al contado a Disvalle Ltda. de Cali, por $5.342.000 con descuento de
5%, pago con cheque.
f) Vende mercancas al contado a Talabartera Castillo Ltda., de Armenia, por un valor de
$1.839.000.
g) Compra mercancas a crdito a Marquetera Ruz & Ca. de Pereira, por $876,400.
h) Devuelve la mercanca comprada en el literal anterior porque no corresponde al pedido.
Se pide:
Registre por los sistemas de inventario permanente las mismas operaciones realizadas por
la empresa Comercial Andina, dadas por el problema anterior. Utilice, cuando sea necesario, 62,5%
como factor de costo

12- Registre en tarjetas krdex, utilizando los tres mtodos de valuacin de inventarios, las
siguientes operaciones realizadas al contado por la empresa Jabones El Oso de la ciudad de
Barranquilla. Inventario inicial: 30.000 unidades a $4.000 c/u
a) 03.01._ Compra 50.000 unidades, factura No.830 a $4.500 c/u
b) 05.01._ Vende 20.000 unidades, factura No.910 a $4,000 c/u
c) 07.01._ Vende 20.000 unidades, factura No.020 a $7,000 c/u
d) 10.01._ Vende 40.000 unidades, factura No.021 a $6.800 c/u
e) 12.01._ Devolucin 10.000 unidades vendidas con factura No.021
13- La Ferretera El Yunque compra y vende a crdito lminas corrugadas de acero Ref. BG4830 y
cdigo: 14350522. Registre los asientos por el sistema de inventario permanente, elabore las
tarjetas de krdex por el mtodo UEPS.
a) 01.10._ Inventario inicial: 3 unidades a $720,000 c/u
b) 03.10._ Compra 7 unidades, factura No.850, a $750,000 c/u
c) 05.10._ Vende 4 unidades, factura No.150 a $1,200,000 c/u
d) 07.10._ Devuelve 2 unidades compradas segn factura No.580
e) 10.10._ Vende 4 unidades, factura No.151, por $1,260,000
14- Registre los asientos por el sistema de inventario permanente y elabore las tarjetas de krdex,
utilizando el mtodo Promedio ponderado, los movimientos de mercancas que se relacionan a
continuacin: radio marca Fnix, referencia: E-10, cdigo: 14350520, localizacin: vitrina No.20,
mximo: 20, mnimo: 3, proveedor: Distrison de Cali, telfono 9645020
a) 01.07._ Inventario inicial: 15 unidades a $20,000 c/u
b) 05.07._ Compra, factura No.450, por 1o unidades a $22,000 c/u
c) 07.07._ Venta, factura No.120, por 5 unidades a $28,000 c/u
d) 10.07._ Compra, factura No.830, por 12 unidades a $25,000 c/u
e) 15.07._ Venta, factura No.121, por 3 unidades a $30,000 c/u
f) 20.07._ Compra, factura No.850, por 8 unidades a $26,500 c/u
g) 25.07._ Devolucin, compra, factura No.850, por 2 unidades
h) 28.07._ Venta, factura No.122, por 25 unidades a $32,000 c/u

Para ver trabajos similares o recibir informacin semanal sobre nuevas publicaciones, visite www.monografias.com

www.monografias.com

i)

30.07._ Devolucin venta, factura No.122, por 7 unidades

15- La empresa XYZ emplea el mtodo de valuacin de inventario peridico y cierra sus libros cada
31 de Diciembre. En 30 de mayo tuvo un incendio habiendo recuperado tan solo una cantidad
de mercancas valorada en $ 15.000.000. La firma de seguros admite pagarle el 76% del
inventario perdido. Normalmente, la poltica de precios de la empresa se basa en obtener una
ganancia bruta sobre las ventas netas del 40%. Calcular cunto recibir la empresa del seguro
contraincendios si el reclamo se basa en la siguiente informacin:
f) Inventario Inicial de mercancas a 1 de Enero: $ 9.440.000
g) Compras del ao hasta la fecha del incendio: $ 718.820.000
h) Ventas del ao hasta la fecha del incendio: $ 960.000.000
i) Fletes de entrada. $ 24.000.000
j) Devoluciones en ventas: $ 18.900.000
k) Devoluciones en compras: $ 12.600.000
16- En enero 1, la empresa Wendy S:A. tena un inventario de 800 camisas de cuello duro, cuyo
costo unitario era de $ 25.000. Las compras de estas camisas efectuadas durante el ao fueron:
19.02._ 400 unidades a $ 26.000
04.04._ 250 unidades a $ 27.000
09.07._ 180 unidades a $ 27.500
26.09._ 350 unidades a $ 27.500
14.11._ 300 unidades a $ 29.000
La empresa aplica un sistema de inventario permanente. En 31 de diciembre se toma el
inventario fsico arrojando una existencia de 600 camisas.
Cul es el valor que se debe asignar al inventario siguiendo la metodologa
a) peps
b) ueps
Bibliografa
1- AMAT O. (2008). Contabilidad y finanza para no financieros. Bogot: Grupo Planeta.
2- BALLESTEROS N. (1985). Fundamentos de contabilidad. Mxico: Interamericana
3- BEJARANO J. (1999) Contabilidad I. Bogot: Politcnico Grancolombiano.
4- CALVACHE J. (1995). Contabilidad General. Bogot: Unisur
5- CARRILLO G. (1987). Fundamento de contabilidad para profesionales no contadores. Bogot:
Continental.
6- CARVALHO j. (2009). Estado de Resultados. Medelln: Universidad de Medelln
7- CASHING J. (1984) Contabilidad. Mxico: McGraw Hill.
8- CORAL L., GUDIO E.. (2008). Contabilidad Universitaria. 6 Ed. Bogota: McGraw-Hill.
9- DE SOUZA l. (2009). Contabilidade ao alcance de todos. 6 Ed. Curitiba: Juru
10- DELGADO L. (1976). Nueva tcnica moderna de la ciencia contable. Bogota: Ed Kelly.
11- FIERRO A. (2011) Contabilidad del patrimonio. Bogot: ECOE
12- FIERRO A. (2009). Contabilidad de Pasivos. Bogot: ECOE.
13- FIERRO A. (2007) Contabilidad de Activos. Bogot: ECOE
14- GUAJARDO G. (2004) Fundamentos de contabilidad. Bogot: McGraw Hill.
15- GUZMAN A. (2006) Contabilidad financiera. Bogot: Universidad de Rosario.
16- HARGADON B. (1982). Principios de contabilidad. Cali: Ed. Norma.
17- HURTADO A. (1982) Manual de contabilidad comercial. Bogota: Ed Presencia.
18- LARA E. (1983). Primer curso de contabilidad. Mxico:: Trillas
19- LOPES DE SA A. (2009). Pratica e teora da contabilidade geral. Curitiba: Juru
20- LOPES DE SA A. (2008). Fundamentos da contabilidade geral. Curitiba: Juru
21- MARULANDA O. (2005) Contabilidad. Medelln: Centro Nal. de Medios para el Aprendizaje, Unad.
22- MINHACIENDA. Decreto 2649 de 1993. Reglamento General de la Contabilidad.
23- MINHACIENDA. Decreto 2650 de 1993. Plan nico de Cuentas. Bogot.
24- ORTIZ A. (2009). Como aprender y ensear contabilidad. Barranquilla: Educosta
25- ORTIZ A. (2008). Pedagoga problmica, significativa y vivencial. Barranquilla: Educosta.
26- PALLEROLA J. (2011). Aplicaciones contables con Microsoft Excel. Bogot: Ed. de la U.
27- RINCON C. (2009). Contabilidad siglo XXI. Bogot: ECOE.

Para ver trabajos similares o recibir informacin semanal sobre nuevas publicaciones, visite www.monografias.com

www.monografias.com

2829303132-

ROJAS D. (1983) ABC de la contabilidad. Bogota: McGraw Hill.


TELESE M. (2008). Haciendo contabilidad. Bogota: Intermilenio
TELESE M. (2006). Conociendo la contabilidad. Bogota: Intermilenio.
TORRES M. (2006). Contabilidad. Manizalez: Universidad de Caldas.
WARREN C., REEVE J., FESS P..(2000) Contabilidad Financiera, 7a Ed. Mxico: International
Thompson Editor.

Autor:
Samuel Lenidas Prez Grau
sammy975603@gmail.com

Para ver trabajos similares o recibir informacin semanal sobre nuevas publicaciones, visite www.monografias.com

Anda mungkin juga menyukai